Sei sulla pagina 1di 99

PROBABILITY &RANDOM PROCESSES- MA6451 RANDOM VARIABLE UNIT-1

PART-A
RANDOM VARIABLE

1. Check whether the following is a probability density function or not. [A.U M/J 2016]
 ke  x , x  0,
f ( x)  
0 , elsewhere

Solution: To Prove  f ( x)dx  1




x
 ke dx  1
0

 e x 
k   1
 1  0

 k  e  x   1
0

K=1
(2et  1) 4
2. If a random variable has the moment generating function given by M X (t )  ,
81
determine the variance of X. [AU M/J 2016]
Solution:
4
(2et  1)4 (2et  1)4  1 2 t 
M X (t )     e 
81 34 3 3 
This is of the form ,which is the MGF of binomial distribution
Here n=4,p=2/3,q=1/3
Mean=np=8/3
Variance-npq=4.2/3.1/3=8/9
 x2
b
3. If a random variable X is known to a distribution function F ( x)  u ( x)[1  e ],
Where b>0 is a constant. Determine its density function. [A.U N/D 2016]
f ( x )  dxd F (x)
x2 x2
d

6

6
d
 u(x) (1  e
dx
)  (1  e ) u ( x)
dx
2
ex x
f ( x )  u ( x) e 6
b
1
Page

MAILAM ENGINEERING COLLEGE DEPARTMENT OF MATHEMATICS


PROBABILITY &RANDOM PROCESSES- MA6451 RANDOM VARIABLE UNIT-1

1
 3 at x  0

2
4. Find the expected value of the distribution where p ( x )   at x  2 [A.U N/D2016]
3



Solution: E ( X )   xp ( x)  0(1/ 3)  2(2 / 3)  4 / 3
X
5. Check whether the following is a probability density function or not.
 e  x , x  0,
f ( x)   [AU M/J 2015]
0 , elsewhere

Solution: To Prove  f ( x)dx  1




L.H.S   f ( x)dx


 
 x  e x 
e dx   
0  1 0
  e x   
0  (0  1)  1
3
(2 x  x 2 ), 0  x  2
6. A random variable ‘X’ has a p.d.f f ( x )   4 , Find P  X  1 .
 0 , otherwise
Solution: [AU N/D 2015]
3 2
 (2 x  x ), 0  x  2
Given f ( x)   4
 0 , otherwise
2
3
  (2 x  x2 )dx
1
4
2
3  x 2 x3  3  23 13  3 4 2 1
  (2  )    (22  )  (12  )   (  ) 
4  2 3 1 4  3 3  4 3 3 2
7. A random variable X is uniformly distributed between 3 and 15 .Find the variance of X.
[AU N/D 2015]
Solution:
a=3 and b=15
(b  a)2 (15  3)2
Variance    12
2

12 12
Page

MAILAM ENGINEERING COLLEGE DEPARTMENT OF MATHEMATICS


PROBABILITY &RANDOM PROCESSES- MA6451 RANDOM VARIABLE UNIT-1

8.The mean and variance of binomial distribution are 5 and 4 .Determine the distribution
Solution: [AU M/J 2015]
Given Mean  np  5 ..................(1)
Variance  npq  4...................(2)
(2) npq 4
 
(1) np 5
4
q
5
4 1
p  1 
5 4
1
n 5
4
n  20
x n x
 1 4
20C x     , x  0,1,2,3,....n, q  1  p
P ( X  x )  p ( x)   4 3
 0 , otherwise

1
9. A random variable ‘X’ has the density function f ( x)  K 1  x 2 in    x  
Find ‘K’. [AU M/J 20 14]
Solution:
Since f (x ) is a p.d.f., we’ve

 f ( x)dx  1


1
K  1 x 2

dx  1  K tan 1 x 

 1



K tan 1 ()  tan 1 ( )  1 
   1
K     1  K  1  K 
2 2 
10. A random variable ‘X’ has a p.d.f f ( x )  3 x 2 , 0  x  1 , Find ‘k’ such that
P  X  k   0.5
[AU M/J 2014]
Solution:
f ( x)  3x 2 , 0  x  1
Given
P  X  k   0.5
3

1
Page

 f ( x )dx  0.5
b

MAILAM ENGINEERING COLLEGE DEPARTMENT OF MATHEMATICS


PROBABILITY &RANDOM PROCESSES- MA6451 RANDOM VARIABLE UNIT-1

1
2
 3x
b
dx  0.5

1
 x3 
3   0.5
 3 b
1 1
1  k 3   k 3  1   0. 5
2 2
1
k  0.5 3
11.X and Y are i.i.d with variance 2 and 3 .Find the variance of 3X+4Y [A.U M/J 2014]
Solution:
Var ( X )  9 Var ( X )  16 Var ( X
=9(2)+16(3)=66
12.A Continuous random variable X has the random variable X has pdf is given by
f ( x)  
a (1 x )2
0 Find a and P  X  4  [A.U M/J 2014]

Solution:
Since f (x) is the pdf, we have

 f ( x)dx  1

2
ie.,  f ( x)dx  1
0
5
 x2 
a x    1
 2 2
 125   8
a 5    2    1
 3   3
 117 
a 3  1
 3 
1
a
42
(ii) P X  4
4
1
P ( X  4)  f ( x) dx
42 2

4
1  x3 
4

  x  
42  2 2
Page

MAILAM ENGINEERING COLLEGE DEPARTMENT OF MATHEMATICS


PROBABILITY &RANDOM PROCESSES- MA6451 RANDOM VARIABLE UNIT-1

1  64   8 
  4    2  
42  3   3
31
a
63
 x 1  x  1
13. Test whether f ( x)   , can be the pdf of a continuous random variable
 0
[A.U N/D 2014]
Solution:
Since f (x) is the pdf, we have

 f ( x)dx  1

0 0
ie.,  f ( x) dx   f ( x) dx
1 1
0 0
x2 x2 1 1
 dx   dx =   1
1
2 1
2 2 2

14. Define Random variable. [AU N/D 2013]


Solution:
A real-valued function defined on the outcome of a probability experiment is called a random variable.

15. Check whether the following is a probability density function or not.


e  x , x  0,   0
f ( x)   [AU M/J 2012]
 0 , elsewhere

Solution: To Prove:  f ( x)dx  1




L.H.S   f ( x)dx


 
 x  e  x 
  e dx   
0    0
 
  e  x

0  (0  1)  1
 The given function is a p.d.f.
5
Page

MAILAM ENGINEERING COLLEGE DEPARTMENT OF MATHEMATICS


PROBABILITY &RANDOM PROCESSES- MA6451 RANDOM VARIABLE UNIT-1

16 Give the Probability law of Poisson distribution and also its mean and variance.
Solution: [AU N/D 2011]
The Probability law of Poisson distribution is given by
e x
P  X  x   p ( x) 
x!
Mean, E ( X )   and Variance  Var (X )  

17. The cumulative distribution function of the random variable X is given by



 0 , x0
 1 1  1
FX ( x)   x  , 0  x  ; Compute P  X   [AU A/U 2011]
 2 2  4
 1
1, x
 2
Solution:
 1  1 1
P  X    1  P X    1  F  
 4  4 4
 1 1 1 3 1
P X    1      1  
 4 4 2 4 4

18. Let the random variable X denotes the sum of obtained in rolling a pair of fair dice.
Determine the probability mass function of X. [AU A/M 2011]
Solution:
x 2 3 4 5 6 7 8 9 10 11 12
p(x) 1 2 3 4 5 6 5 4 3 2 1
36 36 36 36 36 36 36 36 36 36 36

2
19. If a random variable has the moment generating function given by M X (t )  ,
2t
determine the variance of X. [AU M/J 2011]
Solution:
2
Given M X (t )   2(2  t )1
2t
M !X (t )  2(2  t ) 2
M !X! (t )  4(2  t ) 3
E ( X )  M !X (0)  2(2  0)2
6

 2  22
Page

MAILAM ENGINEERING COLLEGE DEPARTMENT OF MATHEMATICS


PROBABILITY &RANDOM PROCESSES- MA6451 RANDOM VARIABLE UNIT-1

2 1
 
4 2
E ( X 2 )  M !X! (0)  4(2  0)3
4 4 1
 4  2 3   
23 8 2
Var ( X )  E ( X 2 )  E ( X )
2

1 1
 
2 4
1
Var ( X ) 
4
20. What is meant by memoryless property? Which continuous distribution follows this property?
Solution: [AU A/M 2010]
If X is a random variable (discrete or continuous), then for any two positive integers
m and n P X  m  n / X  m   P X  n 
which is the memoryless property.
Exponential distribution follows this property.
1  e  x for x  0
21. If a random variable X has the distribution function F ( X )  
 0 for x  0
Where  is the parameter, then find P 1  X  2  [AU N/D 2010]
Solution:
We know that Pa  X  b   F (b )  F ( a )
P1  X  2   F ( 2)  F (1)
  
 1  e 2  1  e   
P1  X  2  e    e 2
22. Every week the average number of wrong-number phone calls received by a certain mail
Order house is seven. What is the probability that they will receive two wrong calls tomorrow?
Solution: [AU N/D 2010]
7
Given :    1
7
e x
PX  x  
x!
1 2
e 1 e 1
P X  2  
2! 2
23. Obtain the mean for a Geometric distribution. [AU A/M 2010]
Solution:
pet
7

We know that M X (t ) 
Page

(1  qet )

MAILAM ENGINEERING COLLEGE DEPARTMENT OF MATHEMATICS


PROBABILITY &RANDOM PROCESSES- MA6451 RANDOM VARIABLE UNIT-1

d 
Mean, E ( X )   M X (t )t 0 
 dt 
 d  pet  
  t  
 dt  (1  qe )  t 0 
d  p  
   t  
 dt  (e  q )  t  0 
 pe t 
  t 2
 (e  q )  t  0
p 1
E( X  2 
p p
Ce  2 x ; x  0
24. If the density function of X equals f ( x)   Find C . What is P[X>2]?
 0 ; x  0
Solution: [AU A/M 2010]

Wkt  f ( x)dx  1


  
2 x 2 x  e 2 x 
 Ce dx  1  C  e dx  1  C   1 C  2
0 0   2 0
 
2 x  e 2 x  4
P[ X  2]   2e dx  2   e
2   2 2
 x2

f ( x)   3 ,  1  x  2 [AU M/J 2006,2009]
25. Show that the function  0 , otherwise
Solution:
 2 2
x2 1  x3  1
 f ( x ) dx  1 3 dx     8  1  1
3  3  1 9
 The given function is a pdf.

26. Given that the p.d.f of a R.V ‘X’ is f ( x )  Kx , 0  x  1 , Find K and P  X  0.5 
[AU M/J 2005]
Solution:

We know that

 f ( x)dx  1
8
Page

MAILAM ENGINEERING COLLEGE DEPARTMENT OF MATHEMATICS


PROBABILITY &RANDOM PROCESSES- MA6451 RANDOM VARIABLE UNIT-1


P  X  0. 5   f ( x)dx
0.5
1 1

 Kxdx  1
0
  2xdx
0.5
1 1
 x2   x2 
K  1  2 
 2 0  2 1
2

1 1
K  1  1
2 4
3
K 2 
4
27. A random variable ‘X’ has a p.d.f f ( x )  K , 0  x  1 , Find K . [AU N/D 2005]
Solution:
Since f ( x) is a pdf

 f ( x)dx  1

1

 Kdx  1  K 1  0  1
0

 K 1

 2x , 0  x  1 1
28. A random variable ‘X’ has a p.d.f f ( x)   , Find P X   .
 0 , otherwise  2
Solution: [AU M/J 2007]
Given f ( x )  2 x, 0  x  1
1
2
  2xdx
0
1
2
 1  x 2 1 1
P X    2    0 
 2  2 0 4 4
 x
29. For the following density function f ( x )  ae ,    x   , Find the value of ' a' .
Solution: [AU M/J 2006]

 x
Given f ( x )  ae ,    x  
9
Page

MAILAM ENGINEERING COLLEGE DEPARTMENT OF MATHEMATICS


PROBABILITY &RANDOM PROCESSES- MA6451 RANDOM VARIABLE UNIT-1

Given f (x ) is p.d.f   f ( x ) dx  1


x
 ae dx  1



2 a  e  x dx  1  2 a  e  x 

0 1
0

1
2a0  1  1  a 
2

30. For the following c.d.f. [AU N/D 2004]


0 , x0

F ( x)   x , 0  x  1 , find (i ) P ( X  0.2) (ii ) P (0.2  X  0.5)
 1 , x 1

Solution:

(i ) P( X  0.2)  1  P( X  0.2)
 1  F (0.2)  1  0.2  0.8
(ii ) P(0.2  X  0.5)  F (0.5)  F (0.2)
 0.5  0.2  0.3
31 The density function of a random variable ‘X’ is given by
f ( x)  Kx (2  x ) , 0  x  2 Find K.
Solution: [AU N/D 2008]

Given f ( x)  Kx (2  x ) , 0  x  2
f ( x) is a p .d . f

  f ( x)dx  1

2

 Kx(2  x)dx  1
0

2 2
2  2 x 2 x3 
K  (2 x  x )dx  1  K    1
0  2 3 0
 8   8
K  4    0  0   1  K 4    1
 3   3
10

12  8  4
K  1  K   1
Page


 3  3

MAILAM ENGINEERING COLLEGE DEPARTMENT OF MATHEMATICS


PROBABILITY &RANDOM PROCESSES- MA6451 RANDOM VARIABLE UNIT-1

3
K 32
4
2
 3 at x  1
 1
32.Find the MGF for the distribution where f ( x)   at x  2
3
 0, otherwise

Solution:
2 1
Given : f (1)  ; f (2)  ; f (3)  f (4)  .............  0
3 3
MGF of a R.V ‘X’ is given by
M X (t )  E etx  

  etx f ( x)
x 0

 e 0 f (0)  e t f (1)  e 2t f (2)  .....


2 1
 et    e2t    0............
3  3
2 1
 et  e2 t
3 3
et
 MGF is M X (t ) 
3
2  et  
33. If a random variable ‘X’ has the MGF,
2
M X (t )  find the variance of ‘X’. [AU M/J 2007]
2t

Solution:
2
Given M X (t )   2(2  t )1
2t
M X (t )  2(2  t ) 2 (1)  2(2  t )2
!

M !X! (t )  4(2  t )3 (1)


 4(2  t )3
E X   M !X (0)  2(2  0)2
1 1
 2 
4 2
 
E X2  M X (0)  4(2  0) 3
!!
11

4 4 1
Page

  
23 8 2

MAILAM ENGINEERING COLLEGE DEPARTMENT OF MATHEMATICS


PROBABILITY &RANDOM PROCESSES- MA6451 RANDOM VARIABLE UNIT-1

Var ( X )  E ( X 2 )  E ( X )
2

1 1 1
  
2 4 4
e  x , x  0
34. Find the MGF of the distribution given by f ( x)  
 0, elsewhere
Solution: [AU N/D 2007]

tx
M X (t )  e f ( x)dx


  etxe  x dx
0

 
( t ) x  e  (  t ) x 
  e dx    
0   (  t )  0

 t
 e  e0
  

 0  1
 t

 The MGF is M X (t ) 
 t

35. State and prove additive property of binomial distribution.


Solution:
The sum of two binomial variates is not a binomial variate.

Let X and Y be two independent binomial variates with parameter n1 , p1  and
n2 , p2  respectively.
Then M X (t )  q1  p1et  1
n


M Y ( t )  q2  p2 e t  n2

 M X Y (t )  M X (t )  M Y (t ) [ X and Y are independen t R.V ' s ]


 q1  p1e t  qn1
2  p2 e t 
n2

The R.H.S cannot be expressed in the form q  pe  . Hence by uniqueness


t n

theorem of MGF X+Y is not a binomial variate.


12

36. Check whether the following data follow a binomial distribution or not. Mean=3;
Variance=4.
Page

Solution: [AU M/J 2004]

MAILAM ENGINEERING COLLEGE DEPARTMENT OF MATHEMATICS


PROBABILITY &RANDOM PROCESSES- MA6451 RANDOM VARIABLE UNIT-1

Given Mean  np  3 ..................(1)


Variance  npq  3...................(2)
(2) npq 4 1
  1
(1) np 3 3
1
q 1 which is  1 Since q>1 which is not possible (0<q<1). The given data does not
3
follow binomial distribution.
37. If ‘X’ is a random variate following binomial distribution with mean 2.4 and Variance
1.44, find P ( X  5) . [AU M/J 2003]
Solution:
For a binomial distribution,
Mean  np  2.4 ..................(1)
Variance  npq  1.44...................(2)

(2) npq 1.44


   0.6
(1) np 2.4

Substituting p  0.4 in (1) ,


n(0.4)  2.4
2.4
n 6
0.4
Therefore the distribution function is

P( X  5)  1  P( X  5)
 1  P( X  0)  P( X  1)  P( X  2)  P( X  3)  P( X  4)
6C0 (0.4) 0 (0.6) 6  6C1 (0.4)1 (0.6) 5  6C2 ( 0.4) 2 (0.6) 4 
 1  
  6C3 (0.4)3 (0.6) 3  6C4 (0.4) 4 (0.6) 2 
P ( X  5  0.04096

38. With the usual notation find ‘p’ for a binomial random variate ‘X’ if n=6 and if
9 P ( X  4)  P ( X  2) [AU M/J 2004]
Solution:
We know that.
Given 9 P ( X  4)  P ( X  2)
13

9  6C 4 p 4 q 2  6C 2 p 2 q 4
Page

MAILAM ENGINEERING COLLEGE DEPARTMENT OF MATHEMATICS


PROBABILITY &RANDOM PROCESSES- MA6451 RANDOM VARIABLE UNIT-1

9 p2  q2
 1  p 
2

9 p2  1  p 2  2 p
8 p2  2 p 1  0
 2  4  32  2  6 1 1
p   or 
16 16 4 2
1
p   ( not possible )
2
p  0.25 , q  0.75
t
 
8
39. If the MGF of a r.v. X is of the form 0.4e  0.6 . What is the MGF of 3X+2.
Solution: [AU N/D 2007]
 t
Given M X (t )  0.4e  0.6  E e 8
  tX

 MGF of 3 X  2 is given by
 
M 3 X  2 (t )  E e 3 X  2 t
e 2t
E e 
 
3 Xt

 e 2t E e 
X (3t )
[ E (e3 Xt )  M X (t )]
 e 2t 0.4e  0.6
3t 8

40. If 3% of the electric bulbs manufactured by a company are defective, find the probability
3
that in a sample of 100 bulbs exactly 5 bulbs are defective (e  0.0498) .
Solution: [AU N/D 2002]
Let X be the R.V denoting the number of defective electric bulbs.
3
Given P ( a bulb is defective ) 
100
p  0.03
n  100
  np  100  0.03  3
e   x
P(' x' bulbs are defective)  P( X  x) 
x!
P (exactly 5 bulbs are defective )  P( X  5)
e  3 35 0.0498  243
 
5! 120
 0.1008
14
Page

MAILAM ENGINEERING COLLEGE DEPARTMENT OF MATHEMATICS


PROBABILITY &RANDOM PROCESSES- MA6451 RANDOM VARIABLE UNIT-1

 
t 1
41. If the MGF of X is 5  4e , find the distribution of X and c.
Solution: [AU N/D 2004]
Let the geometric distribution be
P ( X  x )  q x p , x  0,1,2,.................
The MGF of geometric distribution is given by
p
M X (t )  ..............................(1)
1  qet
1
 4 
M X (t )  5  4e  
t 1 1
 5 1  e t  ...........................(2)
 5 
Comparing (1) and (2), we get
4 1
q  p [ p  q  1]
5 5
 P ( X  x )  pq x , x  0,1,2,.................
x
 1  4 
   
 5  5 
5
 1  4  45
P ( X  5)     P( X  5) 
 5  5  56

42. If the probability is 0.05 that a certain kind of measuring device will show excessive drift,
what is the probability that the sixth of these measuring devices tested will be the first to show
excessive drift?
Solution:
Let ‘X’ be the R.V denoting the no. of measuring devices to show excessive drift.
Here p  0.05  q  1  0.05  0.95
x6
We know that P ( X  x)  q x 1 p
 (0.05)(0.95) 5
 0.0387
43. Find the moment generating function of uniform distribution.
Solution:

tx
M X (t )  e f ( x ) dx

b
  e tx f ( x)dx
15

a
Page

MAILAM ENGINEERING COLLEGE DEPARTMENT OF MATHEMATICS


PROBABILITY &RANDOM PROCESSES- MA6451 RANDOM VARIABLE UNIT-1

b
1
  e tx dx
a
ba
b
1  e tx 
  
ba  t a
1

(b  a)t

ebt  eat 
 The MGF of uniform distribution is

M X (t ) 
e bt
 e at 
(b  a )t
1
44. The time in hours required a machine is exponentially distributed with parameter   ,
2
what is the probability that the required time (i) exceeds 2 hours (ii) exceeds 5 hours.
Solution:
Let X be the R.V which represents the time to repair the machine. Then the density
function of X
is given by
1
1  x
 x
f ( x )  e  e 2 , x  0
2
 k
(i ) P( X  k )  e
1
 2
2
P ( X  2)  e
 e1
(ii ) P ( X  k )  e  k
1
 5
2
P ( X  5)  e
5

2
e
45.Find the moment generating function of uniform distribution [A.U N/D 2017]
Solution:


M X (t)   etx P (X)
x 0

M X (t)   etx ncx p x q n  x
x 0

46.suppose that the duration X in minutes of lang distance calls from your home follows
16

exponential law with p.d.f


Page

MAILAM ENGINEERING COLLEGE DEPARTMENT OF MATHEMATICS


PROBABILITY &RANDOM PROCESSES- MA6451 RANDOM VARIABLE UNIT-1

  5x
f ( x)   e , x  0, What is P  X  5 [AU N/D 2017]
0 , elsewhere

Solution: To Prove  f ( x)dx  1



5

L.H.S P ( X  5)   f ( x )dx
0
5
  x   5 
5 x 0
 e 5   e 5
e  1 1
  e 5 dx        (e  1)
1 1 1
0      5
 5 0  5 5
47. Prove that the function p(x) is a legitimate probability mass function of a discrete
random variable X. [A.U A/M 2017]
x
2 1 
 , x  0,1, 2..
Where p (x)   3  3 
 0, otherwise

 n
21
 p ( x)    
x 0 3  3 
2
2  1 1 
 1      ...
3  3  3  
1
2  1
 1    1
3  3

17
Page

MAILAM ENGINEERING COLLEGE DEPARTMENT OF MATHEMATICS


PROBABILITY &RANDOM PROCESSES- MA6451 RANDOM VARIABLE UNIT-1

PART-B
ONE DIMENSIONAL DISCRETE RANDOM VARIABLE

1 (a). A random variable X has the following function:


X: 0 1 2 3 4 5 6 7
P(X) 0 K 2K 2K 3K K2 2K 2 7K 2  K
:

To Prove: (a) Find K, [A.UN/D 2010M/J 2012, M/J2014]


(b) Evaluate P[X< 6], P[ X  6]
1
(c) If P[ X  C ]  find the minimum value of C.
2
(d) Evaluate P[1.5  X  4.5 / X  2]
(e) Find P[ X  2], P[ X  3], P[1  X  5] .

Formula:  P ( x )  1.
i 1
i


Solution: (a) We know that  P ( x )  1.
i 1
i

0  K  2 K  2 K  3K  K 2  7 K 2  K  1
10 K 2  9 K  1  0
1
K  1, or K 
10
Since P ( X )  0 the value K=-1 is not permissible
1
Hence we have K  .
10
(b) P[ X  6]  P[ X  6]  P[ X  7]  2 K 2  7 K 2  K
2  7 1  19
     .
100  10 10  100
19 81
P[ X  6]  1  P[ X  6]  1  
100 100
18
Page

MAILAM ENGINEERING COLLEGE DEPARTMENT OF MATHEMATICS


PROBABILITY &RANDOM PROCESSES- MA6451 RANDOM VARIABLE UNIT-1

(c) New table


x 0 1 2 3 4 5 6 7
P(x) 0 1 2 2 3 1 2 17
10 10 10 10 100 100 100

P[ X  x ] 0 1 3 5 8 81 83 100
 1
10 10 10 10 100 100 100
=0.5 0.8  0.

The minimum value of c=4.


P[(1.5  X  4.5)  x  2]
(d) P 1.5  X  4.5 / X  2] 
P( X  2)
P[2  X  4.5]

P( X  2)
P(3)  P(4)

1  [ P(0)  P(1)  P(2)]
2 3

 10 10
1 2
1  [0   ]
10 10
5 5
5
 10  10  .
3 7 7
1
7 10
1 1
(e) (i) p( X  2)  P( X  0)  P( X  1)  0  
10 10
(ii) p( X  3)  1  P( X  3)  1  [P( X  0)  P( X  1)  P( X  2)  P( X  3)]
1 2 2 5 1 1
 1[   ]  1  1 
10 10 10 10 2 2
2 2 3 7
p(1  X  5)  P ( X  2)  P ( X  3)  P ( X  4)    
10 10 10 10
.
19
Page

MAILAM ENGINEERING COLLEGE DEPARTMENT OF MATHEMATICS


PROBABILITY &RANDOM PROCESSES- MA6451 RANDOM VARIABLE UNIT-1

1.(b)The probability function function of random variable X is defined as


P ( X  0)  3C 2 , P ( X  1)  4C  10C 2 , P( X  2)  5C  1, where C  0, and P ( X  r )  0 if r  0,1,2.
find (i) the value of C (iii) The distribution function of X. (iii) The distribution function of X.
(iv) The largest value of X for which F(x) < 1/2. [ A.U M/J 2010]

Given:
P( X  0)  3C 2 , P( X  1)  4C  10C 2 , P( X  2)  5C  1, where C  0, and P ( X  r )  0 if r  0,1,
To Prove:
find (i) the value of C (iii) The distribution function of X. (iii) The distribution function of X.
(iv) The largest value of X for which F(x) < 1/2.
Formula:
r

 p( x )  1
x 0

Solution:
(i) P( X  0)  P( X  1)  P ( X  2)  1
3C 2  4C  10C 2  5C  1  1
 7C 2  9C  1  1  0
7C 2  9C  2  0
(7C  2) (C  1)  0
2
C
7
P ( 0  X  2 | X  0)
(ii) P(0  X  2 | X  0) 
P( X  0)
4C  10C 2

9C  10C 2  1
2  4 
4    10  
7
   49 

2  4 
9    10   1
7  49 
 0.4323
New table:
(iii)
X F ( x)  P ( X  x )
0  4
F (0)  P( X  0)  3C 2  3   0.2449  0.5
 49 
1 4
F (1)  P ( X  0)  p( X  1)  7C 2  4C   0.5
7
2 F (1)  P( X  0)  p( X  1)  P ( X  2)  1
20
Page

MAILAM ENGINEERING COLLEGE DEPARTMENT OF MATHEMATICS


PROBABILITY &RANDOM PROCESSES- MA6451 RANDOM VARIABLE UNIT-1

(iv) From new table


1
F ( x)  for x  0.
2
1
2. (a). The probability function of an infinite discrete distribution is given by P (X = j) = ;
2j
j=1,2,…∞. Verify that the total probability is 1 and find the mean and variance of the
distribution. Find also P(X is even), P(X≥5) and P(X is divisible by 3). [A.U N/D 211]

1
Given : P (X = j) = ; j=1,2,…∞.
2j
To Prove: (i) Mean (ii) variance (iii) P(X is even),(iv)P(X≥5) and P(X is divisible by 3).

Solution:

E ( x)   x j p ( x j )
j 1
2 3
1 1 1
 (1)   (2)   (3)   ...
2 2  2
2
1 1 1 
 1  (2)   (3)   ...
2  2 2 
2
1 1
 1  
2 2
2
  
2
E[X ] = x 2
j p( x j )   x j x j  1px j    x j p( x j )
j 1 j 1 j 1

2 3
 1 1 1 
 12  23   34    ...  2
 2 2 2 
1 2
1 1 1 
 12  2 3   34   ...  2
2  2 2 
3
1   1 
 21   
2   2  
6

Var(X) = E(X2)-[E(X)]2
21

= 6-4 = 2
Page

P[X is even] = P [ X= 2]+ P [ X= 4]+…

MAILAM ENGINEERING COLLEGE DEPARTMENT OF MATHEMATICS


PROBABILITY &RANDOM PROCESSES- MA6451 RANDOM VARIABLE UNIT-1

2 4 6
1 1 1
          ...
2 2 2
1
 1 1
 1    1 
 4 3
(ii) P[X ≥ 5] = P [ X= 5]+ P [ X= 6]+…
5 6
1 1
       ...
2 2
5 2
1  1 
   1     ...
 2    2  
5 1
1  1 1
  1  2   16
2

(i) P[X is divisible by 3] or P[X is multiple of 3]


= P [ X= 3]+ P [ X= 6]+…
3 6
1 1
       ...
 3  3
2 3
1 1 1
        ..
8 8 8
1
 1 1
 1    1 
 8 7
Result E ( X )  2 , E ( X 2 )  2 , P[X is even]=1/3, P[ X is divisible by 3]=1/7
2 (b). If the random variable X takes the values 1,2,3 and 4 such that 2P(X=1) =
3P(X=2) = P(X=3) = 5P(X=4) find the probability distribution and cumulative
distribution function of X. [AU N/D 2012,A/M 2015]

Given: If the random variable X takes the values 1,2,3 and 4 such that 2P(X=1) =
3P(X=2) =P(X=3) = 5P(X=4)
To Prove: C.d.f
n
Formula:  p x   1
i 1
i

Solution::
X is a discrete random variable.
Let 2P(X=1) = 3P(X=2) = P(X=3) = 5P(X=4)= k
22

k
2P(X=1) = k,  P(X=1) =
2
Page

MAILAM ENGINEERING COLLEGE DEPARTMENT OF MATHEMATICS


PROBABILITY &RANDOM PROCESSES- MA6451 RANDOM VARIABLE UNIT-1

k
3P(X=2) = k,  P(X=2) =
3
P(X=3) = k,  P(X=3) = k
k
5P(X=4)= k ,  P(X=4) =
5
n
We know that  px   1
i 1
i

k k k
  k  1
2 3 5
61k
1
30
30
k
61
xi p( x i ) F(X)
1 k 15 15
p(1)= = F(1)= p(1)=
2 61 61
2 k 10 15 10 25
p(2)= = F(2)=F(1)+p(2)= + =
3 61 61 61 61
3 30 25 30 55
p(3) = k = F(3)=F(2)+ p(3)=  
61 61 61 61
4 k 6 55 6
p(4) =  F(4)=F(3)+p(4)= + =
5 61 61 61
61
1
61

3. (a). Consider a discrete r. v. ’X’ with probability function


 1

p( X  x)   x( x  1) , x  1,2,3...
0 othewise
Show that E(X) does not exist even though MGF exist. [ AU N/D 2012]

1
Given: p( x )  ,
x ( x  1)
To Prove: E(X) does not exist even though MGF exist
 
Formula: (i) M X (t )   etx p( x) (ii) E ( X )   xp( x)
x 1 x 1
23
Page

MAILAM ENGINEERING COLLEGE DEPARTMENT OF MATHEMATICS


PROBABILITY &RANDOM PROCESSES- MA6451 RANDOM VARIABLE UNIT-1

1
p( x)  , x  1,2,3,...
x( x  1)

E ( X )   xp( x)
x 1

1
x
x 1 x( x  1)

1

x 1 x  1

1 1 1
    ...
2 3 4
1 1 1
 1  1     ...
2 3 4

1
 1  
x 1 x

1
But 
x 1 x
is a divergent series.  E(X) does not exist and hence no moment exists.

Now, MGF of X is given by



tx
M X (t )  e
x 1
p( x)

e tx
 put z= e t
x 1 x( x  1)

zx

x 1 x( x  1)
z z2 z3
    ...
1 2 2  3 3 4
 1 1 1 1 1
 z 1    z 2     z 3     ...
 2  2 3 3 4
 z2 z3  z z2 z3
 z    ...     ...
 2 3  2 3 4
1  z 2 z3 z4 
  log(1  z )      ...
z2 3 4 
 Formula 
 2 3 
 x  x  x ...   log(1  x )
 2 3 
24
Page

MAILAM ENGINEERING COLLEGE DEPARTMENT OF MATHEMATICS


PROBABILITY &RANDOM PROCESSES- MA6451 RANDOM VARIABLE UNIT-1

1 z 2 z3 
  log(1  z )    z  z    ...
z 2 3 
2 3
1 z z 
  log(1  z )  1   z    ...
z 2 3 
1
  log(1  z )  1   log(1  z )
z
1
  log(1  z )  1  log(1  z ), z  1
z
1 
 1    1 log(1  z ), z  1
z 
 M X (t )  1  (e  t  1) log(1  e t ) , t  0 z  e t

M X (t )  1, for t  0 does not exist for t > 0



1
Result M X (t )  1 , for t  0 does not exist for t > 0 , E(X)  1  
x 1 x

3.(b) If the probability mass function of a random variable X is given by


 1 5 
p( x)  Kx 3 , x  1,2,3,4 ,find the value of k , p   X   / X  1 , Find the mean and
 2 2 
variance
of X.X=x 1 2 3 4
P(X=x) k 8k 27k 64k
[A.U M/J 2015]
3
Given: p( x)  Kx , x  1,2,3,4
 1 5 
To Prove: p   X   / X  1 , Find the mean and variance
 2 2 
Solution:
.X=x 1 2 3 4
P(X=x) k 8k 27k 64k

 px   1
i 1
i

.X=x 1 2 3 4
P(X=x) 1 8 27 64
10 100 100 100
25
Page

MAILAM ENGINEERING COLLEGE DEPARTMENT OF MATHEMATICS


PROBABILITY &RANDOM PROCESSES- MA6451 RANDOM VARIABLE UNIT-1


E( X )   x j p( x j
j 1

3
 1   8   271   256 
 (1)   (2)   (3)   4 .
 100   100   100   100 
354

100

E ( X 2 )   x 2 j p( x j
j 1

 1  2 8  2  27  2  64 
 (1)   ( 2)    (3)    4  .
 100   100   100   100 
1300
E( X 2 ) 
100

Var(X) = E(X2)-[E(X)]2
2
1300 1300 
= -
100 100 2
=0.4684
 1 5 
p   X    P X  1
 1 5  2 2
p   X   / X  1 =  
 2 2   P  X  1
P X  1, X  2  P X  2, X  3, X  4
=
1  P X  1
8
 1 5  P X  2  8
p   X   / X  1   100 
 2 2  1  PX  1 99 99
100
354 1300
Result E (x ) , E( x2 )  , Var(X)=0.4684
100 100
26
Page

MAILAM ENGINEERING COLLEGE DEPARTMENT OF MATHEMATICS


PROBABILITY &RANDOM PROCESSES- MA6451 RANDOM VARIABLE UNIT-1

2. COTINUOUS RANDOM VARIABLE

4. (a) .I f the density function of a continuous random variable X is given by


ax ,0  x  1
a ,1  x  2

f ( x)  
 3a  ax ,2  x  3
 0 , otherwise
(i) Find the value of a
(ii) The cumulative distribution function of X
(iii) If x1,x2 and x3 are 3 independent observations of X. What is the probability that
exactly one of these 3 is greater than 1.5? [A.U. N/D 2007,N/D 2008]

Given:
ax ,0  x  1
a ,1  x  2

f ( x)  
 3a  ax ,2  x  3
 0 , otherwise
To Prove: (i) value of a (ii) F((x) (iii) the probability that exactly one of these 3 is greater
than 1.5?

 x
Formula: (i)

 f ( x)dx  1(ii) F [ x]   f ( x)dx


Proof:
(i)Since f(x) is a p.d.f, then
 3



f ( x )dx  1 i.e.,  f ( x)dx  1
0
1 2 3

 axdx   adx   3a  ax dx  1


0 1 2
1 3
 x2   ax 2 
a    ax1  3ax 
2
 1
 2 0  2 2
1   9a 
a   0  a2  1  9a    6a  2a   1
2   2
a 9a
a  4a  1
2 2
27

1
6a  4a  1  2a  1;  a 
2
Page

(ii) (a) If x<0 then F(x)=0


MAILAM ENGINEERING COLLEGE DEPARTMENT OF MATHEMATICS
PROBABILITY &RANDOM PROCESSES- MA6451 RANDOM VARIABLE UNIT-1

x x x
x
(b) If 0  x  1 then F [ x ]   f ( x)dx  axdx   2dx
 0 0
x
1  x2  1 2 x2
    x
2  2 0 4
  x
0 
4
1 x 1
 x2 
(c) If 1  x  2 then F [ x ]   axdx   adx  a    ax 1x
0 1  2 0
1
1  x2  1 x x 1
    x 1  
2  2 0 2 2 4
1 2 x
(d ) If 2  x  3 then F [ x]   axdx   adx   3a  ax dx
0 1 2
1 2 x
x 1 3 x
=  dx   dx     dx
0
2 1
2 2
2 2
1 x
1  x2  1 2 3 1 x2  3 x2 5
    x 1   x    x 
2  2 0 2 2 2 2 2 2 4 4
1 2 3 x
(e) If x  3 then F [ x ]   axdx   adx   3a  ax dx   f ( x)dx
0 1 2 3
1 2 3
x 1 3 x
=  dx   dx     dx  0
0
2 1
2 2
2 2
1 3
1  x2  1 2 3 1 x2 
    x 1   x   1
2  2 0 2 2 2 2 2
3 2 3
1 3 x 1
(iii) P ( x  1.5)  1.5 f ( x)dx 1.5 2 dx  2  2  2 dx  2
Choosing an X and observing its value can be considered as a trial and X>1.5 can be
considered as a success.
i.e., p=P[X>1.5]=1/2
1 1
 p  ,q 
2 2
As we choose 3 independent observations of X ,n=3.
By Bernoulli’s theorem P(x) = nCxpxqn-x]
P (exactly one value>1.5)= P(1 success)
1 31
28

1 1 3
Result : = 3C1     
2 2 8
Page

MAILAM ENGINEERING COLLEGE DEPARTMENT OF MATHEMATICS


PROBABILITY &RANDOM PROCESSES- MA6451 RANDOM VARIABLE UNIT-1

4.(b).The Distribution F of a continuous random variable X is given by


0 ,x  0
 1
 x2 ,0  x 
 2
F ( x)   [AU N/D 2011]
 1  3 (3  x ) 2 , 1  x  3
 25 2
 1 ,x  3

Find the p,d,f of X and evaluate P(|X|≤1) and P(1/3<X<4) using both PDF and CDF.

Given: F(x)
To Prove: (i) P df of X (ii) P(|X|≤1) a(iii) P(1/3<X<4) using both PDF and CDF.
Solution:
 1
 2x ,0  x 
2

d  6 1
f ( x)  F ( x )   (3  x) ,  x  3
dx  25 2
 0 , otherwise


 3  13
P(|X|≤1)=F[1]-F[-1]= 1  (3  1) 2  -0=
 25  25
1
1 1 1 1
2
6 6  x2  1 6  1   3 1  13
1 f ( x) dx  0  0 2 x dx  1 25 (3  x) dx  x 
2 2
0  
25 
3 x 
2
1    3       
4 25  2   2 8  25
2 2

  1 2 
P(1/3<X<4)= F[4]-F[1/3]= 1    
  3  
1
4 2 3
1 6  1 1  6  9   3 1  8
P(  x  4)   f ( x ) dx   2 x dx   (3  x) dx  0       9       
3 1 1 1 25  4 9  25  2   2 8  9
3 3 2

Result : P(|X|≤1=13/25, P(1/3<X<4)=13/25, P(|X|≤1)=13/25

2
 x
 2
;x  0
5 (a). If f ( x )   xe , then show that f(x) is a pdf and find F(x). [AU N/D 2014]
 0 ; x  0

2
 x
29

 2
;x  0
Given: f ( x)   xe
Page

 0 ; x  0

MAILAM ENGINEERING COLLEGE DEPARTMENT OF MATHEMATICS


PROBABILITY &RANDOM PROCESSES- MA6451 RANDOM VARIABLE UNIT-1

To Prove: (i) P.d.f of x (ii) F(x)



Formula:  f ( x) dx 1


Solution:
 x2 

 xe 2
dx   e t dt   e t  

0
 
  e   e 0  1
0 0

Therefore given f(x) is PDF.


x2
x x2 2 x2
 
2 t 2
Result: F(x)=P(X≤ x)=  xe dx   e dt  1 e , x≥0.
0 0

5. (b) The cumulative distribution function (cdf)of a random variable X is F(x)=1-(1+x)e-x


,x>0.Find the probability density function of X. Mean and Variance of X.
[AU M/J 2006, N/D 2010]
-x
Given: F(x) =1-(1+x)e ,x>0

To Prove: (i) f(x) (ii) E(X) (iii) Var(x)



Formula: (i) E ( x )   xf ( x ) dx (II) VAR(x)=E(X2)-E(x)2
0

P.d..f f(x) = [ ( )]
d
 [1  e  x  xe  x ]  e  x  [ x (e  x )  e  x ]  xe  x , x  0
dx
 
E[x]=  xf ( x) dx   x 2 e  x dx
0 0

  ex   ex   e  x 
 x2    ( 2 x )  2 
 2  3 
2
  1   (1)   (1)   0
 
E[x2]=  x 2 f ( x) dx   x 3 e  x dx
0 0

  ex  2  e
x
  ex   e x 
 x3    (3 x )  2 
 ( 6 x )  3
 ( 6 )  3 
6
  1   (1)   (1)   (1)   0
Var(X)=E(x2)-[E(x)]2=6 – 4 = 2
Result
f ( x )  xe  x , E(x)=2,Var(x)=2
30
Page

MAILAM ENGINEERING COLLEGE DEPARTMENT OF MATHEMATICS


PROBABILITY &RANDOM PROCESSES- MA6451 RANDOM VARIABLE UNIT-1

6(a) A continuous random variable X has a pdf f(x)=kx2e-x , x≥0. Find K, rth moment of x
about orgin mean and variance. [AUM/J2013]

Given: f(x)= kx2e-x , x≥0

To Prove: (i) Find K ,(ii) E[x] (iii) Var(x)



Formula:  f ( x)dx  1

 
1
 kx
2
e dx  1  k  x 2 e  x dx  1  2k  1  k 
x

0 0
2
To find Mean:
 
1
Mean=E[x]=  xf ( x ) dx   x 3 e  x dx
0 0
2

1  e x  2  e
x
  e x   e x  1
 x3    ( 3 x )  2
 ( 6 x )  3
 ( 6 )  3 
 ( 6)  3
2   1   (1)   (1)   (1)   0 2
To find Variance:
 
2 1
E[x ]=  x f ( x) dx   x 4 e  x dx
2

0 0
2

1  e x   ex  2  e
x
  ex   e  x 
  x 4    (4 x 3 ) 2 
 (12 x )  3
 ( 24 x )  3
 24  4 
 12
2   1   (1)   (1)   (1)   (1)   0
Var(X)=E(x2)-[E(x)]2=12 – 9 = 3.
1
Re sult k  , E(X)=3, Var(x)=12
2
6 (b). A continuous random variable X has the pdf
K
f ( x)  ;    x   find
1  x2
(i) The value of K,(ii) Distribution function of X, (iii)P[X≥0]
K
Given: f ( x)  ; x
1 x2
To Find: (i) The value of K,(ii) Distribution function of X, (iii)P[X≥0]
 x
Formula: (i)  f ( x)dx  1

(ii) F [ x ]   f ( x)dx 


Solution:
31
Page

MAILAM ENGINEERING COLLEGE DEPARTMENT OF MATHEMATICS


PROBABILITY &RANDOM PROCESSES- MA6451 RANDOM VARIABLE UNIT-1


(i)W.k.t  f ( x)dx  1


K   
1 x 2

dx  1  k tan 1 ( x)  
  
 1  k tan 1 ( )  tan 1 (  )  1  K     1
 2 2
1
K    1 K 

(ii)Distribution function of X
x x
1 1 1 1 1  1 
F [ x]   f ( x )dx  
 1 x 2
dx  tan 1 x

 x
 

 
tan 1 ( x)  tan 1 (  ) 
  tan ( x)  2 
   
(iii)To find P[X≥0]
 
1 1 1 1 1   1
P[ X  0]   f ( x )dx  
 1 x 2


  
dx  tan 1 x 0  tan 1 ()  tan 1 (0)    0 
 2

0 0  2
1 1  1 
Result: K  , F(X)=  tan ( x )  , P[ X  0]  1 / 2
  2 

7. (a) The probability density function of a random variable X is given by


x ; 0  X 1

f X ( X )  k ( 2  x);1  X  2 (i) Find the value of K? (ii) Find P0.2  X  1.2
0 ; otherwise

(iii) What is P0.5  X  1.5 / X  1 (iv) Find the distribution function of f(x)
[A.U. A/M 2011]
x ; 0  X 1

Given: f X ( X )  k (2  x );1  X  2
0 ; otherwise

To Prove: (i) Find the value of K? (ii) Find P0.2  X  1.2 (iii) What is
P0.5  X  1.5 / X  1
(iv) Find the distribution function of f(x)
 x
Formula: (i)  f ( x )dx  1 (ii) F ( x )  P  X  x    f ( x) dx
 0

Solution:
Since f (x) is the pdf, we have

 f (x)dx1

32

2
ie.,  f ( x)dx  1
Page

MAILAM ENGINEERING COLLEGE DEPARTMENT OF MATHEMATICS


PROBABILITY &RANDOM PROCESSES- MA6451 RANDOM VARIABLE UNIT-1

1 2

 x dx   k (2  x ) dx  1
0 1
1 2
 x2   x2 
   k  2 x   1
 2 0  2 1
1   1 
 2  0  k (4  2)  (2  2 )  1

1  1
k   1
2 2
1  1
k  
2 2
k 1
(ii) P0.2  X  1.2
1.2
P 0.2  X  1.2    f ( x ) dx
0.2
1 1.2
  x dx   (2  x )dx
0.2 1
1 1.2
 x2   x2 
    2 x  
 2  0.2  2 1
 1 0.04   1 
    (2.4  0.72)  (2  )
2 2   2 
 0.48  1.68  1.5
 0.66

(iii) P0.5  X  1.5 / X  1


P0.5  X  1.5)  ( X  1
P0.5  X  1.5 / X  1 
P( X  1)
P1  X  1.5

P( X  1)
1.5
P 1  X  1.5   ( 2  x) dx
1
1.5
 x2 
 2 x  
 2 1
33

 (1.5) 2 1 
 2(1.5)   (2  )
2 2 
Page

MAILAM ENGINEERING COLLEGE DEPARTMENT OF MATHEMATICS


PROBABILITY &RANDOM PROCESSES- MA6451 RANDOM VARIABLE UNIT-1

 3  1.125  1.5
 0.375
2
P  X  1   ( 2  x) dx
1
2
 x2 
 2 x  
 2 1
 1 
  4  2  (2  )
 2 
 0. 5
0.375
P 0.5  X  1.5 / X  1   0.75
0.5
x
(iv) F ( x )  P  X  x    f ( x) dx
0

(i) if x  0 then F ( x)  0
x
(ii) If 0  x  1 then F ( x )   x dx
0
x
 x2  x2
  
 2 0 2
1 x
(iii) If 1  x  2 then F ( x )   x dx   ( 2  x) dx
0 1
1 x
 x2   x2 
    2 x  
 2 0  2 1
1  x2 1 
    ( 2 x  )  ( 2  ) 
2  2 2 

1 x2 1 x2
  2x   2   2x  1
2 2 2 2
x
(iv) If x  2 then F ( x)   f ( x) dx

1 2 x
  x dx   ( 2  x ) dx   0 dx
0 1 2
1 2
 x2   x2 
34

    2 x  
 2 0  2 1
Page

MAILAM ENGINEERING COLLEGE DEPARTMENT OF MATHEMATICS


PROBABILITY &RANDOM PROCESSES- MA6451 RANDOM VARIABLE UNIT-1

1  1 
  4  2  ( 2  ) 
2  2 
1
Result: P 0.2  X  1.2   0.6 , P 0.5  X  1.5 / X  1 =0.375

7 (b). The cumulative distribution function of the random variable X is given by


 0 , x0

FX ( x)  a (1  x) ,  1  x  1 ; [A.U A/M 2015]
 1 , x  1

 1
Find the value of a Find P  X   , P  0.5  X  0 
 4
 0 , x0

Given: FX ( x )  a (1  x) ,  1  x  1
 1 , x 1

 1
To Prove: (i) P  X   (ii) P  0.5  X  0 
 4

Formula: (i)  f ( x)dx  1




Solution:
0 ,x0
d 
f ( x)  F ( x )  a ,1  x  1
dx  0
 , otherwise


 f ( x)dx  1
1

 a dx  1
1 a(1  1)  1
1
ax 1 1 2a  1
1
a
2
 1 1
P  X    P[  X  1]
 4 4
35
Page

MAILAM ENGINEERING COLLEGE DEPARTMENT OF MATHEMATICS


PROBABILITY &RANDOM PROCESSES- MA6451 RANDOM VARIABLE UNIT-1

1
1
 dx
1 2
4

1
 [ x]11
2 4
3

8
0
1
P 0.5  X  0   2dx
1

2

1 1
 [ x ]0 1 
2 2 4
1  1 1
Result a  , P X   
2  4 4
8.(a) A continous random variable X that can assume any value between x=2, x=5 has a
f ( x)  k (1  x) .Find P[ X  4] [A.U A/M 2015]
density.

Given: f ( x )  k (1  x ) ,x=2,x=5
To prove: P[ X  4]

Formula:  f ( x)dx  1


Solution:

 f (x)dx1

5
ie.,  K (1  x))dx  1
2

5
 x2 
k x    1
 2 2

 35 8 
k   1
 2 2
2
k
36

27
Page

MAILAM ENGINEERING COLLEGE DEPARTMENT OF MATHEMATICS


PROBABILITY &RANDOM PROCESSES- MA6451 RANDOM VARIABLE UNIT-1

4
(ii ) P X  4   f ( x) dx
2
4
  k (1  x ) dx
2
4
2
 (1  x) dx
2
27
4
2  x2 
 x  
27  2 2
2 16
 [12  4] 
27 27
2
Result k  P X  4   16 / 27
27

3. MOMENT GENERATING FUNCTION


1
8 (b). The probability function of an infinite discrete distribution is given by P(X=x)=,
2x
x=1,2,…...  Find the mean and variance of the distribution. Also find P(X is even).
1
Given: P(X=x)= x , x=1,2,…... 
2
To Prove: E  X  ,Var(X)

Formula: M X (t )  e tx
 
p ( x) , E  X   Mean  M X 0  , Variance  E X 2  E  X 
' 2

x 1

tx
Proof: We know that M X (t )  e
x 1
p( x)

1
  e tx 
x 1 2x
x

 et 
   
x 1  2 
2
et  et 
     ........
2 2
2
e t  e t  et  
 1      ........
2  2 2 

1
37

et  et 
 1  
2  2
Page

MAILAM ENGINEERING COLLEGE DEPARTMENT OF MATHEMATICS


PROBABILITY &RANDOM PROCESSES- MA6451 RANDOM VARIABLE UNIT-1


1
  e tx 
x 1 2x
x
 et


   
x 1  2 
2
et  et 
     ........
2 2
2
et  et  et  
 1      ........
2  2 2 

1
et  et 
 1  
2  2
2
et  et 
     ........
2 2

et  et  et 2 
 1      ........ [Using
2  2 2 
1
et  et 
 1  
2  2
(1  x) 1  1  x  x 2  .....]


et 2  et   1

2 2 1
et
M X (t ) 
2  et
 (2  e t ) 1 e t ...(1)
' t t 2 t t 1 t
M X (t )   e ( 2  e ) (  e )  ( 2  e ) e
1
 e 2t (2  e t )  2  (2  e t ) e t ...2

M '' X t   2 2  e t 
2
   e 
e 2t  e 2t  2 2  e t
3 t


 2  et  1
et  e  12  e   e 
t t 2 t
...3
'
Now E  X   Mean  M X 0 
 11  2  put t  0 in 2 
 
E X 2  M X 0   6
''
Put t  0 in 3
'
38

Mean 1  2
 
Variance  E X 2  E  X  =6-4=2
2
Page

MAILAM ENGINEERING COLLEGE DEPARTMENT OF MATHEMATICS


PROBABILITY &RANDOM PROCESSES- MA6451 RANDOM VARIABLE UNIT-1

Now P(X=even)=P(X=2)+P(X=4)+…....
2 4
1 1
       ....
2 2
2
1
 
 2
 2
1
1  
 2
 a 
 Sum to  of a goemetric series is 1  r , a 1 term, r  common ratio 
st

1
 4
1
1
4
1 4
 
4 4 1
1

3

Result

MGF Mean Variance P(X=even)

2 2 1

et 2  e 
t 1 3

 1  X3
 , x0
9 a). Let ‘X’ be a random variable with p.d.f f ( x )   3 e
0 , othewise
Find (a) P(X>3) (b) MGF of ‘X’ (c) E(X) and Var(X)
 1  X3
 , x0
Given : f ( x)   3 e
0 , othewise
To Prove: a) P(X>3) (b) MGF of ‘X’ (c) E(X) and Var(X)

Formula: (i) M X t   E (e tX )
39
Page

MAILAM ENGINEERING COLLEGE DEPARTMENT OF MATHEMATICS


PROBABILITY &RANDOM PROCESSES- MA6451 RANDOM VARIABLE UNIT-1

 1  13

Solution: Given f ( x)   3 e , x  0
0 , othewise

P(X > 3)   f  x  dx
3
 x
1 3
 e dx
3 3

 x 
1 e 3 
  
3 1

 3  3
  0  e 1 
 e 1
1
  0.3679
e
(b) To find MGF of ‘X’
M X t   E (e tX )

  e tx f  x  dx

 x x
1 3  1 3 
  e tx e dx  x  0, f  x   e 
o
3  3 
  1
1  t  3  x
 e dx
3 0
 1 
1  t  x
  e  3  dx
30

  1 t  x 
 
1  e 3  
  
3   1 
   3  t  
 0
40
Page

MAILAM ENGINEERING COLLEGE DEPARTMENT OF MATHEMATICS


PROBABILITY &RANDOM PROCESSES- MA6451 RANDOM VARIABLE UNIT-1

 
1  1 
  
3   1  3t  
  3  
 
 
1  1 
  0   e 
0 
3 1 
   t
  3 
1
MGF of X  M X (t ) 
1  3t
(c) To find E(X), Var(X):
1 1 MG Mea Var
M X t    1  3t 
1  3t F n ianc
2
M X' t   1  3t   3 e

 31  3t 
2

E ( X )  Mean  M X' 0  3


M X'' t   61  3t   3  181  3t 3
3

M X'' 0   18

E ( X 2 )  M X'' 0  18
  
Var  X   E X 2  E  X 
2

 18  3 2  18  U sin g ( A) and
 Var  X   9
P(X>3)

1 1 3 9
 0.3679
e 1  3t

 1  2x

9. (b). Let ‘X’ be a random variable with p.d.f f ( x )   2 e , x  0 Find mean and
0 , othewise
variance [ A.U N/D 2012, 2013 R.P]
41

Given: f(x)
Page

To Prove: E(X), Var(X):

MAILAM ENGINEERING COLLEGE DEPARTMENT OF MATHEMATICS


PROBABILITY &RANDOM PROCESSES- MA6451 RANDOM VARIABLE UNIT-1


Formula: M X (t )   e tx f  x  dx , E ( X )  Mean  M X' 0 , Var  X   E  X 2   E  X 
2



Solution:
(b) To find MGF of ‘X’
M X t   E (e tX )

  e tx f  x  dx

 x x
1 2  1 3 
  e tx e d  x  0, f  x   e 
o 2  3 
  1
1  t  x
  e  2  dx
20
 1 
1  t  x
  e  2  dx
20

  1 t  x 
 
1  e 2  
  
2   1 
   2  t  
 0
 
1  1 
 
2   1  2t  
  2  
 
 
1  1 
  0 
2   1 
 e 
0
  2  t  
 
1
MGF of X  M X (t ) 
1  2t
(c) To find E(X), Var(X): 42
Page

MAILAM ENGINEERING COLLEGE DEPARTMENT OF MATHEMATICS


PROBABILITY &RANDOM PROCESSES- MA6451 RANDOM VARIABLE UNIT-1

1 1
MX t  1 2t
12t
2
MX t  1 2t 2
'

2
21 2t
E(X)  Mean MX' 0  2 .....(A)
3 3
MX'' t  412t  2 81 2t
MX'' 0 8

E(X2)  MX'' 0 8 ....B


  
Var X  E X2 E X
2

8 22 8  4 Using(A)andB
 Result Var X  4, E(X)  Mean MX' 0  2
 x  2x
 , x0
10 (a). Let ‘X’ be a random variable with p.d.f f ( x)   4 e
0 , othewise
Find mean and variance [A.U N/D 2010,2012 2013,M/J 2014 ]

Given: f (x)
To find: mean and variance

Formula: M X (t )   e tx f  x  dx


MX t E(etX)

 etx f x dx

 x x
Solution: 1   1 3 
etx e 2dx 
 x 0, f  x  e 
o 4  3 
 1 
1  tx
 e 2  xdx
40


 1  1 

  t 
2
   t  x 
2
1 e   e   
 x
43

 (1) 2
4 1  1  
    t     t  
Page

  2   2   0

MAILAM ENGINEERING COLLEGE DEPARTMENT OF MATHEMATICS


PROBABILITY &RANDOM PROCESSES- MA6451 RANDOM VARIABLE UNIT-1

2
 1 
MGF of X  M X (t )   
 1  2t 
MGF of X  M X (t )  1  2(2t )  3(2t ) 2  4(2t ) 4  .....
t t2 t3 t4
MGF of X  M X (t )  1 ( 4)  ( 24 )  (192)  . .(1920 )...
1! 2! 3! 4!
Result 1,2,3,4 moments=4, 24,192,1920
 2 e 2 x , x  0
10.(b) A random variable has the p.d.f given by f  x   
0 , x  0
Find (a) the moment generating function [A.U N/D 2014]

(b) The first four moment about the orgin.

Given: f(x)

To Prove: a) the moment generating function (b) The first four moment about the orgin.


Formula: (i) M X t    e f x  dx
tx



Solution:
f (x) 2e2x, x 0 ....1

M X t    e f x  dx
tx



  e tx  2e  2 x dx U sin g 1, x  0
o

 2 e 2t x dx
0

 e   2 t  x 
 2 
  2  t   0
 e  ax 
U sin g e
 ax
dx  . Here a  2  t 
 a 
2

2t

 e   e 0 
2
 0  1  2
44

2t 2t
 MGF is M X t   22  t 
1
Page

MAILAM ENGINEERING COLLEGE DEPARTMENT OF MATHEMATICS


PROBABILITY &RANDOM PROCESSES- MA6451 RANDOM VARIABLE UNIT-1

2
M X' t   22  t   1
2
 22  t  .....1A
2 1
1s moment M X' 0    Put t  0 in 1A
4 2
M x'' t   2 2 2  t   1
3

3
 42  t  .....2

4 1
2 nd moment M X'' t    Put t  0 in 2
8 2
M X''' t   122  t   1
4

4
 122  t  .....3

12 3
3rd moment M X''' 0   Put t  0 in 3
16 4
M XIV t   482  t   1
5

5
 482  t  ....4 

48 3
4th moment M XIV t    Put t  0 in 4
32 2

MGF 1st moment 2 nd moment 3rd moment 4th moment


2 1 1 3 3
2t 2 2 4 2

11 (a). Find the mean, variance and MGF of the random variable X having the pdf
x ,0  x  1

f ( x )   2  x ,1  x  2
0 , otherwise

[ AU N/D 2013]
Given: f(x)
To Prove: mean, variance

Formula: E ( X )   x f ( x)dx


Solution:
45


Mean= E(x)=  x f ( x)dx
Page



MAILAM ENGINEERING COLLEGE DEPARTMENT OF MATHEMATICS


PROBABILITY &RANDOM PROCESSES- MA6451 RANDOM VARIABLE UNIT-1

1 2 1 2
 x3   2x 2 x3  13   8   2 1 
  x (x ) dx   x( 2  x) dx           0    4         1
0 1  3 0  2 3 1  3   3   2 3 
 2
2
E(x )= x

f ( x)dx

1 2 1 2
2 2  x4   2x3 x 4  1   16 16   2 1  7
  x (x ) dx   x ( 2  x) dx           0           
0 1  4 0  3 4 1  4   3 4   3 4  6

Var(X)=E(x2)-[E(x)]2=(7/6)-1=1/6.

MGF=  e tx f ( x) dx


1 2 1 2
tx tx  xe tx e tx   ( 2  x )e tx e tx 
  e x dx   e ( 2  x ) dx    2    2
0 1  t t  0  t t 1
 e t e t   0 1 2t
   0 e   e
t
e t  1 t
   2     2 
     2     2   2 e  1
  
2

 t t   t t    t t   t t   t

Result E(X)=1,VAR(X)=I/6

5. DISTRIBUTIONS
BINOMIAL DISTRIBUTION
11(b)Define binomial distribution, Find the moment generating function of binomial
distribution and also find its mean and variance. [A.U M/J 14]
Solution:
A random variable X is said to follow binomial distribution if it assumes only non-
negative values and its probability mass function p(x) is given by
nC p x q n  x , x  0,1,2,3,....n, q  1  p
P ( X  x)  p ( x)   x
 0 , otherwise

Where n and p are parameters.


To find MGF
We know that the moment generating function of a random variable X about origin
whose probability function p(x) is given by
M X (t )  E e tX  
n
  e tx p( x)
46

x 0
n
  e tx nC x p x q n  x
Page

x 0

MAILAM ENGINEERING COLLEGE DEPARTMENT OF MATHEMATICS


PROBABILITY &RANDOM PROCESSES- MA6451 RANDOM VARIABLE UNIT-1

n
  ( pet ) x nC x q n  x
x 0
n
  nC x ( pet ) x q n x
x 0

 q n  nC1q n 1 pet  nC2 q n 2 pet   1


 2
 .....................

 q  pet 
n


 M X (t )  q  pet n

To find Mean and Variance:



M X (t )  q  pet  n

 M 1X (t )  n q  pet   n 1
pet
Put t=0, we get
n 1
M 1X (0)  nq  p  p
M 11
X (t )  np q  pe
t
  n1 t
e  e t (n  1)(q  pet ) n2 pet 
Put t=0, we get
X (0)  np q  p 
M 11  n 1
 (n  1)(q  p) n2 p 
X (0)  np1  ( n  1) p   np  n p  np
M 11 2 2 2

 n 2 p 2  np(1  p)
 n 2 p 2  npq
Variance M 11 1
X (0)  M X (0)  2

 npq  n 2 p 2  n 2 p 2 Var ( X )  npq

POISSON DISTRIBUTION
12(a).By Calculating the moment generating function of Poisson distribution with parameter
 , prove that the mean and variance of the Poisson distribution are equal.
Solution:
e x
P  X  x   p ( x)  [ A.U.A/M 2011,N/D 2014,M/J 2015,N/D2015]
x!

e   x
tx

tx
The m.g.f M X (t )   e p ( x)   e
x 0 x 0 x!
e   ( e t ) x
 
( e t ) x
47


 e 
x 0 x! x 0 x!
Page

MAILAM ENGINEERING COLLEGE DEPARTMENT OF MATHEMATICS


PROBABILITY &RANDOM PROCESSES- MA6451 RANDOM VARIABLE UNIT-1

  e t ( e t ) 2 
 e   1    ....... 
 1! 2! 
 e   ee  e  e 1
t t

d 
Mean, E ( X )   dt M X (t )
 t 0
d
 dt
  d
 t  0  dt
t
 
  e e 1    ee e   
t

t 0
 

 e  

d e t 
dt
 
t
e   e   e e  e t
t  0
 
t 0

 e   e   

 
E X2
d
 dt
 d
 t  0  dt
t 
  M X (t )   e   e e et 
 t 0
 

 e  

d e t t 
dt
e e 
t 0
 
 t t

  e   [ e e e t  e t  e t e  e ] t  0   e   [ e    e  ]
2
  
Var(X)  E ( X 2 )  E ( X )    2  2  
2

GEOMETRIC DISTRIBUTION

Definition of geometric distribution:


A discrete random variable ‘X’ is said to follow geometric distribution, if it
assumes only non-negative values and its probability mass function is given by
x 1
P ( X  x )  p ( x)  q p , x  1,2,3,... 0  p  1 where q  1  p

12(b) Find the moment generating function, mean and variance of geometric distribution?
Solution::
The moment generating function of geometric distribution is given by
M X (t )  E (e tx )

  e tx p ( x )
x 0

  e tx q x1 p
48

x 1

p  tx x qx
q x 1
Page

 e q 
q x1 q

MAILAM ENGINEERING COLLEGE DEPARTMENT OF MATHEMATICS


PROBABILITY &RANDOM PROCESSES- MA6451 RANDOM VARIABLE UNIT-1

p 
  (qe t ) x
q x 1


p t
q

qe  qet    qe 
2 t 3
 ... 

p t
q
1
qe 1  qe t  qet     2
 ... 

 pet 1  qet  1 1
 1  x   1  x  x 2  .....
pe t
M X (t ) 

1  qe t 
pe t
Mean:WKT, M X (t ) 
 1  qe t 
' d
M X (t )  ( M X (t ))
dt
d pe t


dt 1  qe t 
 (1  qet )e t  e t (qet ) 
p  
 1  qet
2
  
 e t  qe 2t  qe 2t 
p  
 1  qe
t 2
  
 et 
p 2 
                 (1)
 1  qe
t
  
Putting t=0 in (1)
'  p  p 1
 E ( X )  M X ( 0)   2 
 2  1  q  p
 1  q   p p

'' d2
MX (t )  2 ( M X (t ))
dt
d '
 ( M X (t ))
dt
d  pe t 
  
dt  1  qe t 2 
49

 (1  qet ) 2 e t  e t 2(1  qet )(qet ) 


p  
 
Page

4
 1  qet 

MAILAM ENGINEERING COLLEGE DEPARTMENT OF MATHEMATICS


PROBABILITY &RANDOM PROCESSES- MA6451 RANDOM VARIABLE UNIT-1

''
 (1  qe 0 ) 2 e 0  e 0 2(1  qe 0 )(qe 0 ) 
E ( X 2 )  M X (0)  p  
 1  qe 0 4
 
 (1  q ) 2  2(1  q )( q) 
p  
 1  q 4 
 p 2  2 pq 
p   1  q  p
 p4 
 p( p  2q) 
 
 p3 
 p  2q 
 2 
 p 
 p  q  q  1  q 
 2  2   p  q 1
 p   p 
1 q
E(X 2 )  2  2
p p
1 q 1
Var ( X )  E ( X 2 )  E ( X ) 
2
2
 2  2
p p p
CONTINOUS DISTRIBUTION
UNIFORM DISTRIBUTION
13(a) Find the moment generating function of Uniform distribution. Hence find its mean
and variance.
Solution:
1
f ( x)  ,a  x  b
ba
b b b
tx 1 tx1  etx 
The mgf M X (t )   e f ( x)dx   e dx   
a a
ba b  a  t a
1  ebx  e ax  ebt  e at
  
ba t  (b  a )t
b
   x r f ( x)dx
1
r where f ( x) is p.d . f of ' X '............(1)
a
b
1
   xf ( x)dx
1 [ put r  1 in (1)]
50

a
Page

MAILAM ENGINEERING COLLEGE DEPARTMENT OF MATHEMATICS


PROBABILITY &RANDOM PROCESSES- MA6451 RANDOM VARIABLE UNIT-1

b b
1 1  x2 
x dx 
a
ba b  a  2  a


1
b2  a2 b  ab  a  
2(b  a) 2b  a 

Mean 
b  a 
2
b
 21   x 2 f ( x)dx [ put r  2 in (1)]
a

b b
1 1  x3 
2
x dx 
a
ba b  a  3  a


1 3 3
b a 
b  a  b 2  ab  a 2
   
3(b  a ) 3b  a 


b 2
 ab  a 2 
3
 Variance   12  11   2


b 2
 ab  a 2  b  a 
 
 2

3  2 


b 2
 ab  a 2

b 2  2ab  a 2   
3 4


2

4 b  ab  a  3 b  ab  a 2
2 2
  
12
4b 2  4ab  4a 2  3b 2  6ab  3a 2

12


b 2
 2ab  a 2

b  a  2

12 12

 Variance 
b  a2
12
13(b) Define exponential distribution, Find the moment generating function of exponential
distribution and also find its mean and variance. [A.U M/J 2015]
Solution:
A continuous random variable ‘X’ is said to follow a exponential distribution with
parameter
51

  0 if its probability density function is given by


Page

MAILAM ENGINEERING COLLEGE DEPARTMENT OF MATHEMATICS


PROBABILITY &RANDOM PROCESSES- MA6451 RANDOM VARIABLE UNIT-1

 e   x , x0
f ( x)  
 0, otherwise
To find MGF:

M X (t )   e tx f ( x)dx
0
 
x
  e e tx
dx    e (  t ) x dx
0 0

 e    t  x  
    e   e 0  
    t   0    t 


  t 

 M X (t ) 
  t 

To find Mean and Variance:


 1
M X (t )  
  t  1  t

1
 t
 1  
 
t t2 tr
1  2  .........  r  ........
  
2
t  t  2!   t r  r! 
 1     2   .........    r   ........
  2!     r!   
t 1
 Mean 11  coefficient of 
1! 
t2 2
 12  coefficient of  2
2! 
Variance  12  11   2

2 1 1
2
 2  2
  
1
Variance  2

52
Page

MAILAM ENGINEERING COLLEGE DEPARTMENT OF MATHEMATICS


PROBABILITY &RANDOM PROCESSES- MA6451 RANDOM VARIABLE UNIT-1

GAMMA DISTRIBUTION

14(a) Define Gamma distribution, Find the MGF of Gamma distribution and also find mean
and variance. [A.U N/D 2014,M/J 14,0 2013,2011 ,N/D 2016,N/D2017]
Solution:
The continuous random variable ‘X’ is said to follow a Gamma distribution with parameter
 if its probability density function is given by
 e  x x  1
 ,   0,0  x  
f ( x )    ( )
 0, otherwise

To find MGF:
M X (t )  E e tx  

  e tx f ( x)dx
0

e  x x  1
  e tx dx
0
(  )

1
  e tx e  x x  1 dx
 ( ) 0

1
  e  (1t ) x x  1 dx
 ( ) 0
put (1  t ) x  u If x  0, u  0
(1  t )dx  du If x  , u  
  1
1  u   du 
  e u    
 ( ) 0 1 t  1 t 

1 u  1e u
( ) 0 1  t  1 1  t 
 du


1 u  1e  u
( ) 0 1  t 
 du


1  1 u

1  t  ( ) 
 
u e du
0

1
  ( )  (n)   x n 1e  x dx
1  t  ( )
53

1
Page


1  t 
MAILAM ENGINEERING COLLEGE DEPARTMENT OF MATHEMATICS
PROBABILITY &RANDOM PROCESSES- MA6451 RANDOM VARIABLE UNIT-1


M X (t )  1  t  , t  1

To find Mean and Variance:



M X (t )  1  t 
  1
M 1X (t )   1  t  (1)
Mean  11  M 1X (0)  
  2
X (t )   (  1)1  t 
M 11 (1)
 21  M 11
X (0)   (  1)

Variance   12  11   2

  (  1)  2  2    2
Variance  

NORMAL DISTRIBUTION
Defn:
1  x  2
1 2   
 
A normal distribution is a continuous distribution given by y  e where X is a
 2
1  x  2
1 2   
 
continuous normal variate distributed with density function f ( x )  e with mean
 2
 and standard deviation  .
14(b).Find the moment generating function, mean and variance of normal distribution?
[A.U N/D 2014]

M X (t )  E (e tX )  e
tX
f ( x ) dx

2
 ( x )
1 2  2
  e tX e dx
  2
2
 ( x )
1 tX 2  2
 e e dx
 2 

x
Put z  ,  dz  dx

2
 ( z ) 
1 t  z    2
 e e dz
 2 
 ( z 2 2 t  z )
e t
54

2
  e dz
2 
Page

MAILAM ENGINEERING COLLEGE DEPARTMENT OF MATHEMATICS


PROBABILITY &RANDOM PROCESSES- MA6451 RANDOM VARIABLE UNIT-1

 1 1
e t 2
( z  t ) 2
 ( t )
2
2

  e dz
2 
t 2 2
 t  1
2 2
e 2
(z t )
  e dz
2  
Put u  z  t , du  dz
t 2 2
 t  1
2 2
e (u )
  e2 du
2 
t 2 2
 t  1
2 2
e (u)
 2   e2 du  2
2 
t 2 2
 t
2
e

MEAN:
d 
E ( X )   M X (t ) 
 dt  t 0
2 2
d   t 
t 
2
 e 
dt   t  0
2 2
  t t  
 e 2
  t 2   
  t  0

d2 
E ( X )   2 M X (t ) 
2

 dt  t 0
2 2
d   t 
t 
 e
dt 
2
  t   2

 t  0
2 2 2 2
   t t    t t  

    t 2  2
e 2
e 2
2    2  2
   
t 0

Var ( X )  E ( X 2 )  E ( X )
2

 2  2  2   2
55
Page

MAILAM ENGINEERING COLLEGE DEPARTMENT OF MATHEMATICS


PROBABILITY &RANDOM PROCESSES- MA6451 RANDOM VARIABLE UNIT-1

15(a) Memoryless Property of Exponential Distribution:


If X is exponentially distributed then P ( X  S  t / X  S )  P ( X  t ), for any S , t  0
Proof:

P ( X  K )    e   x dx
K

 e  x 
P ( X  K )   
   k
 e  K               (1)
P( X  S  t and X  S )
P( X  S  t / X  S ) 
P( X  S )
P( X  S  t ) e   ( S  t )
   S by (1)
P( X  S ) e
 e   t  P( X  t )
P( X  S  t / X  S )  P( X  t )
The converse of this result is also true.
(i.e.,) If P ( X  S  t / X  S )  P ( X  t ) then X follows an exponential distribution.
15(b) State and prove the memoryless property of the geometric distribution?
[A.U M/J 2016, N/D 2015]
Solution:n:
Let ‘X’ be the random variable following a geometric distribution.
The memoryless property is given by P X  s  t / X  s   P( X  t ) for any s, t  0
Consider

P X  s  t    pq x 1

x  s  t 1

 
 p q s  t  1 1  q s  t  2 1  ....
 p q s t

 q s  t  1  ....

 p q s  t 1  q  .q 2  ...
 p q s  t 1  q  1
p q s t
          (1)
1 q
p qs
 P( X  s )              (2)
1 q
p qt
56

and P( X  t )           (3)
1 q
Page

Hence

MAILAM ENGINEERING COLLEGE DEPARTMENT OF MATHEMATICS


PROBABILITY &RANDOM PROCESSES- MA6451 RANDOM VARIABLE UNIT-1

P( X  s  t and X  s)
 P( X  s  t / X  s ) 
P( X  s )
P( X  s  t )

P( X  s )
p q s t 1 q
 x by (1) and (2)
1 q pq s
q s t

qs
 qt
pqt p p
   1
1 q 1 q p
P( X  s  t / X  s)  P ( X  t ) by (3)
16(a) Derive probability mass function of Poisson distribution as a limiting case of binomial
[A.U N/D 2014, 2013 R.P]
distribution.
Solution:
(i) The number of trails ‘n’ should be indefinitely large. [i.e., n tends to infinity]
(ii) The probability of successes ‘p’ for each trail is indefinitely small.
(iii) np   , should be finite where  is a cons tan t
We know that the binomial distribution is
P( X  x)  nC x p x q n x
n!
 p x (1  p) n x [ q  1  p]
(n  x)! x!
1.2.3........(n  x)(n  x  1)...........n x (1  p) n
 p
1.2.3.............(n  x) x! (1  p) x
x
1.2.3........(n  x )(n  x  1)...........n  p 
   (1  p) n
1.2.3.............(n  x) x!  (1  p) 
x
  
(n  1)(n  2)........(n  x  1)  n    
n
 

x!
1  
    n  p  n 
1  
 n
n
(n  1)(n  2)........(n  x  1)  x 1  
57

 1  
x! n   x
x
 n
1  
Page

 n

MAILAM ENGINEERING COLLEGE DEPARTMENT OF MATHEMATICS


PROBABILITY &RANDOM PROCESSES- MA6451 RANDOM VARIABLE UNIT-1

nx
(n  1)(n  2)........(n  x  1)  x  
 1  
x! nx  n
 1  2    x  1 
11  1  ................1    n x
 n  n    n  x   
  1  
x!  n
When n  
n x
Lt
  
we know that 1    e
n   n 
Lt  1  Lt  2  Lt   x  1  
1    1    ..........  1     1
n   n  n   n  n     n  

e   x
 P( X  x )  , x  0,1,2,..........., 
x!
Hence the probability mass function of a random variable ‘X’ which follows Poisson
distribution is given by
 e  x
 x  0,1,2,..........., 
P( X  x)  p( x)   x! ,
 0, otherwise

6.BASED ON SUMS OF DISTRIBUTIONS


16(b) The time (in hours) required to repairs a machine is exponential distributed with
1
Parameter   . [AU N/D 2009]
2
(i) What is the probability that the repair time exceeds 2 2hours?
(ii) What is the conditional probability that a repair takes atleast 10h given that its
duration exceeds 9h?
Solution:
1
Given:  
2
If X represents the time to repair the machine , the density function of X is given by
Formula:
f ( x )  e  x
x
 x 1
f ( x)  e  e2, x0
2
58
Page

MAILAM ENGINEERING COLLEGE DEPARTMENT OF MATHEMATICS


PROBABILITY &RANDOM PROCESSES- MA6451 RANDOM VARIABLE UNIT-1


 x 
 x  x
1 1 1 e 2 
(i ) p ( X  2)   e 2 dx   e 2 dx   
2
2 22 2  1 
 2  2

 x 
  e 2   e 1  0.3679
 2
(ii ) p( X  10 / X  9)  p( X  1) (by the memoryless property)

 x 
 x  x
1 2 1 1 e 2 
  e dx   e 2 dx   
1
2 21 2  1 
 2 1

 x 
  e 2   e  0.5  0.6065
(i ) p ( X  1) ,  1

Result:
(i ) p( X  2)  0.3679 (ii ) p( X  1)  0.6065

17(a) Suppose that telephone calls arriving at a particular switchboard follow a Poisson
process with an average of 5 calls coming per minute . What is the probability that up to a
minute will elapse unit 2 calls have come into the switch board? [AU A/M 2011]
Solution:
Given: The Poisson process applies with time until 2 Poisson events following a Gamma
1
distribution with   and   2 .
5
Let the random variable X be the time in minutes that transpires before 2 calls come.

To prove:
1
Px  1  25 xe 5 x dx
0
1 1
  e 5 x   e 5 x    x 5 x 1 5 x 
 25 x    (1)    25 e  e 
  5   25   0  5 25 0
 1 1   1   6 1
 25  e 5  e 5     0    25 e 5  
 5 25   25   25 25 
Px  1  1  6e 5  0.96
59

Result:
Page

MAILAM ENGINEERING COLLEGE DEPARTMENT OF MATHEMATICS


PROBABILITY &RANDOM PROCESSES- MA6451 RANDOM VARIABLE UNIT-1

17.(b)Trains arrive at a station at 15 minutes intervals starting at 4a.m.If a passenger arrive at


a station at a time that uniformly distributed between 9.00 and 9.30, find the probability that
he has to wait for the train for (1) less than 6 minutes (2) more than 10 minutes.
[A.U. N/D 2014 P.Q.T]

1
Solution: Formula: f  x   0  x  30
ba
Given:
1
f x 
30  0
1
f x 
30
1) p x  6 Starting time x  4
6
1
p4  x  6    dx
4
30
1
 [ x ]64
30
1
 [ 6  4]
30
1
 [ 2]
30
1

15
2) p x  10  p10  x  30
30
 1 
 
10
  dx
 30 
1 30
 [ x ]10
30
1
 [30  10]
30
1 2
 [20] 
30 3
Result:
p 4  x  6  = 1/15 p  x  10   2 / 3
18(a)The marks obtained by a number of students for a certain subject is assumed to be
60

normailly distributed with mean 65 and S.D of 5.If 3 students are taken at random from this
set, what is the probability that exactly 2 of them will have marks over 70
Page

[ A.U A/M 2010,2011]

MAILAM ENGINEERING COLLEGE DEPARTMENT OF MATHEMATICS


PROBABILITY &RANDOM PROCESSES- MA6451 RANDOM VARIABLE UNIT-1

Solution: X Denote marks obtained by the given set of students


Given:   65,   5
X 
Formula: WKT, Z 

70  65
When X  70, z  1
5
 P ( X  70)  P (0  Z  )
 0.5  P( 0  Z  1) =0.5-0.0.3413=0.1587
Result P ( X  70)  0.1587
18(b) transcendental Meditation(TM) is a continuous random variable X normally distributed
with mean 37.6 cc/mm and S.D 4.6 cc/min. Determine the probability that during a period of
T.M a person’s oxygen consumption will be reduced by (i) atleast 44.5 cc/min (ii) atmost
35.0 cc/min (iii) Anywhere from 30.0 to 40.0 cc/min [A.U. N/D 2012]
Solution:
X 
Given   37.6,   4.6, Z 

X  37.6

4. 6
To find P X  44.5 PX  35 P30  X  40 
(i) For atleast 44.5 cc/min
(i.e)., X  44 .5
44.5  37.6
 Z  1.5
4.6
P X  44.5  PZ  1.5
 0.5  P Z  1.5
 0.5  0.4332
 0.068
(ii) For atmost 35.0 cc/min
(i.e)., X  35
35  37.6
 Z  0.5652
4. 6
P X  35  P Z  0.5652 
 0 .5  0 .2157
 0.2843
(iii) Anywhere from 30.0 to 40.0 cc/min
30  37.6
(i.e)., X 1  30  Z1   1.6521
61

4.6
Page

MAILAM ENGINEERING COLLEGE DEPARTMENT OF MATHEMATICS


PROBABILITY &RANDOM PROCESSES- MA6451 RANDOM VARIABLE UNIT-1

40  37.6
X 2  40  Z2   0.52173
4. 6

P30  X  40   P 1.6521  Z  0.52173


 0.4505  0 .1985
 0.6490
1
Result: P X  44.5 =0.068, PX  35 =0.2843 , P30  X  40  =1
19(a). Let X and Y be independent normal variates with mean 45 and 44 and standard
deviation 2 and 1.5 respectively. What is the probability that randomly chosen values of X and
Y differ by 1.5 or more?
Solution: [ A.U N/D 2011]
Given: X is N 45,2 and Y is N 44,1.5 )

Therefore by property of additive U  X  Y follows the distribution N 1, 4  2.25 
i.e., N 1,2.5
P X and Y differ by 1.5 or more  P X  Y  1.5  P U  1.5
 1  P U  1.5
 1  P  1.5  U  1.5
  1.5  1 U  1 1.5  1
 1  P  
 2.5 2.5 2.5 
 1  P  1  Z  0.2
 1  0.3413  0.0793
Result  0.5794
19(b) If X is a uniform random variable in the interval (-2,2) find the probability density
function
Y | X | AND E[Y ] [P A.U N/D 2011]
Solution:
Given:
X ~ U [ 2, 2]
 1
 ; 2 x 2
Formula: The pdf f ( x)   b  a
0 ; otherwise
1
 ; 2 x2
f ( x)   4
62

0 ; otherwise
Page

Given Y | X |

MAILAM ENGINEERING COLLEGE DEPARTMENT OF MATHEMATICS


PROBABILITY &RANDOM PROCESSES- MA6451 RANDOM VARIABLE UNIT-1

C.D.F of G(Y )  PY  y   P| x |  y 


 F  y   F ( y )
y y
1 1
  dx   dx
2
4 2
4
1 y 1 y
 x2  x2
4 4
1 1
  y  2   y  2
4 4
1
 y
2
1
dG ( y )  ; 0  y  2
p.d . f . g ( y )   2
dy 0 ; otherwise
2
1
E (Y )   y 2 dy
0
2
1  y2  4
    1
2  2 0 4
Result: E (Y ) =1
20(a). If the probability that an applicant for a driver’s license will pass the road test on any
given trial is 0.8. what is the probability that he will finally pass the test (1) on the fourth trial
and (2) In less tha 4 trials? [PRP A.U A/M 2010,2015]

Solution:
Let X denote the number of trials required to achieve the first success. Then X is a geometric
distribution given by P( X  r )  q r 1 p; r  1,2,3,...
Here p  0.8 & q  0.2
(1) P( X  4)  0.8(0.2) 41  0.8(0.008)  0.0064
3
(2) P( X  4)   0.8(0.2) r 1  0.8(1  0.2  0.04)  0.992
r 1

20(b). Find the MGF of the two parameter exponential distribution whose density function
is given by f ( x)   e   ( x  a ) , x  a and hence find the mean and variance?
[A.U A/M 2010]
Solution:
63

Given :
f ( x)   e   ( x  a ) , x  a
Page

MAILAM ENGINEERING COLLEGE DEPARTMENT OF MATHEMATICS


PROBABILITY &RANDOM PROCESSES- MA6451 RANDOM VARIABLE UNIT-1


tx
Formula : M X (t )  e f ( x) dx
The MGF 


M X (t )   e tx  e  ( x  a ) dx
a

  e a  e tx e  x dx
a

  e a  e (  t ) x dx
a

a  e (  t ) x 
 e  
  (  t )  a
 e  (  t ) a 
  e a 0  
  (  t ) 
 e at

 t
d 
Mean E ( X )   M X (t )
 dt t0
 d   e at  
  
 dt    t   t  0
   at at    
  ae  e  2
( 1) 
   t   (  t )   t 0
  a e at  e at  a   1
         a 
  t (  t ) 2  t 0   ( ) 2  
 d 2 
E ( X 2 )   2 M X (t )
 dt t 0
d    at at   
   ae  e  2

dt    t   (  t )  t  0
   at at    a       2  
  ae ( a)  e  2
( 1)  ae at  2
  e at  
3 
(  1) 
   t   (  t )   (  t )   (  t )   t 0
    a      2  
  a 2     a
2  2
   
3 
    (  )   ( )   ( )  
64

 a a 2 
Page

 a 2    2 
    

MAILAM ENGINEERING COLLEGE DEPARTMENT OF MATHEMATICS


PROBABILITY &RANDOM PROCESSES- MA6451 RANDOM VARIABLE UNIT-1

Var ( X )  E ( X 2 )  E ( X )
2

2
2a 2  1
 a2   2  a  
   
2a 2  2 1 2a  1 1
 a2   2  a  2    2 Var ( X )  2
       , 

21 (a). The marks obtained by a number of students in a certain subject are assumed to be
normally distributed with mean 65 and standard deviation 5. If 3 students are selected at
random from this group, what is the probability that two of them will have marks over 70?
[A.U A/M 2010]
Solution: Given:   65,   5,
X 
Formula: Z 

c
When X  70
70  65
 Z 1
5
P X  70   P1  Z   
 0.5  P 0  Z  1
 0.5  0.3413
 0.1587 N  3, P  0.1587, q  1  p  0.8413 PY  2  3C 2 p 2 q
 3(0.1587 ) 2 (0.8413) =  0.0636
Result: P X  70  =0.0638

21 (b). X is uniformly distributed with mean 1 and variance 4/3, find P X  0


[A.UM/J 2015]

Given: Mean 1,variance 4/3


To Prove: P X  0
Solution: We are given that mean=1
ba
 1 ………………………(1)
2
Variance=4/3
b  a 24

……………………(2)
12 3
65

From (1),(2) b=3,a=-1


Page

MAILAM ENGINEERING COLLEGE DEPARTMENT OF MATHEMATICS


PROBABILITY &RANDOM PROCESSES- MA6451 RANDOM VARIABLE UNIT-1

1 1
f ( x)  
ba 4
0
1 1 1
P( X  0)   dx  [ x]10  Result P ( X  0) =1/4
1
4 4 4
22 (a). If X is a uniform random variable in the interval (-1,1) find the probability density
 x 
function Y  sin   [ A.U N/D 2011]
 2
1
 ; 1 x 1
Given: The pdf f ( x)   2
0 ; otherwise
 x  x
F (Y )  PY  y   P sin  y   P(  sin 1 y )
 2  2
2
 P( X  sin 1 y )

2
sin 1 y )
 2
1 sin 1 y )
  dx  [ x] 1

2
1 2
 [ sin 1 y  1]
2 

Range  1  y  1

 1
dF ( y )  ;1  y  1
p.d . f . g ( y )    1  y 2
dy 
0 ; otherwise
2
1
E (Y )   y 2 dy
0
2
1  y2  4
    1
2  2 0 4
Result : E(Y )=1
2(1  x ) 0  x  1
22 (b). If the pdf of X is given by f ( x)   find its rth moment.
 0 otherwise
Hence evaluate E[(2 X  1) 2 ] [ A.U N/D 2012]
66

Given f(x)
Page

To Prove: (i) rth moment (ii) E[(2 X  1) 2 ]

MAILAM ENGINEERING COLLEGE DEPARTMENT OF MATHEMATICS


PROBABILITY &RANDOM PROCESSES- MA6451 RANDOM VARIABLE UNIT-1

b
Formula E{ X )   x r f ( x) dx
r

s
1
E{ X r )   r 2( x r  x r 1 ) dx
0

1 1
 x r 1 x r  2 
 2   
0 r  1 r  20
 1 1 
 2  
 r  1 r  2  
2

r  1 r  2
2 1
E(X )  
1  1 1  2 3
2 1
E( X 2 )  
2  1 2  2  6
E (2 X  1)  E[4 X 2  1  4 X ]
2

 4 E[ X 2 ]  E[1]  4 E[ X ]
1 1
 4( )  1  4   3
6  3
1
E{ X r )   x r 2(1  x) dx
0

Result E ( 2 X  1) =3, E ( X ) =1/3

23(a) A Random variable X is uniformly distributed over (0.10) find p  x  3 , p  x.  7  ,


p  x  7  , p 2  x  5  [A.U M/J 2013]

1
Given : f  x  
10
To find: p  x  3 , p  x.  7  , p  x  7  , p 2  x  5 
1
Solution: f  x   0  x  30
ba
1
f x 
10  0
1
f x 
67

10
Page

MAILAM ENGINEERING COLLEGE DEPARTMENT OF MATHEMATICS


PROBABILITY &RANDOM PROCESSES- MA6451 RANDOM VARIABLE UNIT-1

3
1 1 3 3
i) p x  3   dx  x0 
0
10 10 10

10
1 1 10 3
(ii) p x  7    dx  x 7 
7
10 10 10
5
1
p2  x  5   dx
2
10
1
 [ x]52
10
1 3
 [5  2] 
10 10

Result (i) p  x  3 =3/10 , p x  7   3 / 10 , p 2  x  5  iii) p x  7 0

23 (b) An office has four phones lines .Each is busy about 10% of the time .Assume that the
phone lines act independently (1) What is the probability that all four phones are busy
(ii) What is the probability that atleast two of them are busy. [ A.U M/J 2013]

Given: x=4,p=0.1,q=0.9

To Prove: 1) What is the probability that all four phones are busy (ii) What is the
probability that atleast two of them are busy.
Formula: P( X  x)  q x 1 p; x  1,2,3,...
Solution:

Using geometric distribution with x=4,p=0.1,q=0.9


given by P( X  x )  q x 1 p; x  1,2,3,...

(1) P( X  4)  0.1 (0.9) 41  0.1(0.9) 3  0.0729


P( X  2)  1  px  2
(2) P ( X  1)  1  px  1
 1  (0.1)(0.9)11  0.9
Result: P ( X  4) =0.0729,0.9
68

24(a). If the probability that an applicant for a driver’s license will pass the road test on any
given trial is 0.8. what is the probability that he will finally pass the test (1) on the fourth trial
Page

and (2) In less tha 4 trials? [A.U A/M 2010,2015]

MAILAM ENGINEERING COLLEGE DEPARTMENT OF MATHEMATICS


PROBABILITY &RANDOM PROCESSES- MA6451 RANDOM VARIABLE UNIT-1

Solution:
Given: Here p  0.8 & q  0.2
Let X denote the number of trials required to achieve the first success.
Then X is a geometric distribution given by P( X  r )  q r 1 p; r  1,2,3,...
Here p  0.8 & q  0.2
(1) P( X  4)  0.8(0.2) 41  0.8(0.008)  0.0064
3
(2) P( X  4)   0.8(0.2) r 1  0.8(1  0.2  0.04)  0.992
r 1

Result:
P ( X  4 )= 0.0064, P ( X  4) =0.992
25(a).In a certain city, the daily consumption of electric power in millions of Kilowatt-
hours can be considered as a random variable following gamma distribution with
1
parameters   and   3 . If the power plant in this city has a daily capacity of 12
2
million kilowatt-hours, what is the probability that this supply of power will be insufficient
on any given day? [A.U M/J 2016]
Solution:
Let X denotes the daily consumption of power in millions of kilowatt-hours. Given X
1
follows gamma distribution with parameters   and   3 .
2
The probability density function of X is given by

e  x (x) 1
f ( x)  ,x  0

x 2
1 2 x
e   x

2  2   1 e 2 x2
3 16

Probability of insufficient supply  P [ X  12]

  x
1 
  f ( x ) dx   e 2
x 2 dx
12
16 12


 x x x 
1  2e 2 e 2 e 2 
 x  2x 2
16   1  1
2
 
 1 
3
 
69

 2 2 2  12
Page

MAILAM ENGINEERING COLLEGE DEPARTMENT OF MATHEMATICS


PROBABILITY &RANDOM PROCESSES- MA6451 RANDOM VARIABLE UNIT-1


1  x x x
  2 x 2 e 2  8 xe 2  16e 2 
16   12


1  x2

16 
e 
 2 x 2  8 x  16 
 12


1 x

  e 2 x 2  4 x  8 
8   12

1

8

0  e  6 144  48  8 
6
RESULT:  e  25  0.002478  25  0.06195

26(a).A coin is biased so that a head is twice as likely to appear as a tall. If the coin is tossed
6 times, find the probabilities of getting (1) exactly 2 heads, (2) at least 3 heads, (3)
atmost 4 heads. [A.U M/J 2016]
Solution:
2
(I) Getting a head is success. Since the coin is biased, Probability of success p 
3
2 1
q  1 
3 3
The coin is tossed 6 times. n  6
Let X denotes the number of heads is 6.
1
 X is a binomial random variate with parameter n  6  q  .
3
x 6 x
 2 1
Binomial Distribution is P [ X  x]  6C x     , x  0,1,2,...., 6
 3   3
2 6 2
2 1
P [ X  2]  6C 2      15(0.44444)(0.01235)  0.0823
 3 3
P [ X  3]  1  P[ X  3]
 1  P[ X  0]  .....  P[ X  2]
 1  6 1
 2 1
5 21
 2   1 
 1     6C1      6C 2     
 3   3   3  3   3  
 1 12 60 
 1     0.8998
 36 36 36 
70

P[ X  4]  1  P[ X  4]
 1  P( X  5)  P ( X  6)
Page

MAILAM ENGINEERING COLLEGE DEPARTMENT OF MATHEMATICS


PROBABILITY &RANDOM PROCESSES- MA6451 RANDOM VARIABLE UNIT-1

5 6
  2 1  2 
 1  6C 5      6C 6   
  3   3  3  
RESULT:
256
1  0.6488
729

26(b).The length of time a person speaks over phone follows exponential distribution with
mean 6 mins. What is the probability that the person will talk for(1) more than 8
mins, (2)between 4 and 8 mins. [A.U M/J 2016]
Solution:
Let the random variable X denotes the length of time (in minutes) a person speaks.
Given X is exponential with mean = 6 mins.
1 1
6 
 6
1 x
Pdf of X is f ( x)  e x  e 6 , x  0
6
1 4
 8 
6 3
(i) P [ X  8]  e e  0.2636
8 8 x
1 
(ii) P [ 4  X  8]   f ( x) dx   e 6 dx  0.2498
4 4
6

27(a).If the probability of success is 1/100 , how many trials are necessary in order that the
probability of atleast one success is greater than 1/2? [A.U N/D 2016]
Solution:

Given: p  1 / 100
q  1  p  1  1 / 100  99 / 100
P ( hitting atleast one)  p( x  1)
 1  p ( x  0)
 1  (.99) n  1 / 2
RESULT: n  69
27(b).A and B shoot independently until each has his own target. The probability of their
hitting the target at each shot is 3/5 and 5/7 respectively? Find the probability that B will
require more shots than A. [A.U N/D 2016]
Solution:
Given: p  3 / 5
71

q  1 p 1 3/ 5  2 /5
p ( x  r )  pq r 1
Page

MAILAM ENGINEERING COLLEGE DEPARTMENT OF MATHEMATICS


PROBABILITY &RANDOM PROCESSES- MA6451 RANDOM VARIABLE UNIT-1

p( x  r )  (3 / 5)(2 / 5) r 1
p  5/ 7  q 1 p 1 5/ 7  2 / 7
p( y  r )  (5 / 7)(2 / 7) r 1

Probability that B will require more shots than A



  p[ x  r and y  r  1, r  2,............]
r 0

  p ( x  r )  p[ ( y  r  1) or y  (r  2)............]
r 0

r 1
  3 / 7 4 / 35 2 / 7 7 / 5
r 1

RESULT  6 / 31

28(a).If log e X is normally distributed with mean 1 and variance 4, find P (1 / 2  X  2)


given that log e 2  0.693 [A.U N/D 2016]
Solution:
Given: Mean   1
Variance  2  4    2
Let L  log e X
L   L  1 log e X  1
The standard normal variate z   
 2 2
1
log e 1
When X=1/2 , z1  2
2
log 1  log 2  1

2
 log 2  1  .0693  1
 
2 2
z1  0.8465
log e 2  1 0.693  1
When X  2 , z2    0.1535
2 2

P1 / 2  X  2  P z 1  z  z 2 
 P 0.8465  z  0.1535
72
Page

 ( Area between z  0 and z  0.8465)   Area between z  0 and z  0.1535 

MAILAM ENGINEERING COLLEGE DEPARTMENT OF MATHEMATICS


PROBABILITY &RANDOM PROCESSES- MA6451 RANDOM VARIABLE UNIT-1

(Using Normal table)

Result =0.2996-.0596=0.24 (approx)


28(b). In a normal distribution 31% of items are under 45 and 8% of items are over 64.
Find the mean and the standard deviation of the distribution. .[A.U N/D 2015]
Solution:

X 
We know that Z 

Let Z  Z1 where X  45 and

Z  Z 2 when X  64

P(0  Z  Z1 )  0.31

P( Z1  Z  0)  0.19

From tables, Z1   0.49

45  
i.e.,   0.49 (OR) 45     0.49   (1)

P( Z  Z1 )  0.8 (OR) P(0  Z  Z 2 )  0.42

From tables, Z 2  1.40

64  
i.e.,  1.40

64    1.40  ( 2)

(1)  ( 2)   19  1.89

 19
  10
 1.89

From (1),   45  (0.49  10)  45  4.9  49.9  50

 Mean  50 and S .D.  10


73

   50 and   10
Page

MAILAM ENGINEERING COLLEGE DEPARTMENT OF MATHEMATICS


PROBABILITY &RANDOM PROCESSES- MA6451 RANDOM VARIABLE UNIT-1

 2 x,0  x  b
29(a). The p .d. f of a random variable X is given by f ( x ) . For what value of
0, otherwise
b is f(x) a valid pdf? Also find the cdf of the random variable X with the above pdf.

[A.U N/D 2015]


Solution:
Formula

 f ( x ) dx  1

b

 2 xdx  1
0
b
 x2 
2   1
 2 0
b2  1
b 1
x x xb
 x2 
F ( x )   f ( x ) dx   2 x dx  2    x 2
 0  2 0

 0 , if x  0

F ( x )   x 2 if 0  x  1
 1, if x  1

 0 , if x  0

RESULT: F ( x )   x 2 if 0  x  1
 1, if x  1

29 b) i) The number of monthly breakdowns of a computer is a random variable having a

Poission distribution with mean equal to 1.8.Find the probability that this computer
Will function for a month (1) Without a break down(2) With only one breakdown.
(A.U N/D 2017)
Solution:
Given : mean=   1.8
74

Let X denote the no. of breakdown of a computer


Page

MAILAM ENGINEERING COLLEGE DEPARTMENT OF MATHEMATICS


PROBABILITY &RANDOM PROCESSES- MA6451 RANDOM VARIABLE UNIT-1

e   x
The probability of distribution is P( x  x ) 
x!

e 1.8 1.8
x
P( x  x) 
x!

e 1.8 1.8
0
(1) P(Without a break down) = P ( x  0)   e 1.8  0.1653
0!
e 1.8 1.8
1
(2) P(With only one break down) = P ( x  1)   e 1.8 (1.8)  0.2975
1!

30(a) If the probability that a target is destroyed on any one shot is 0.5. What is the
probability that it would be destroyed on 6 th attempt? (A.U N/D 2017)
Solution:
Given : p  0.5; q  1  0.5  0.5

FORMULA P( X  x)  q x 1 . p
5 6
6 1 1 1 1 1
P ( X  6)  q . p     
2 2  2 64.

30(b) Suppose X has an exponential distribution with mean equal to 10. Determine the

value of x such that P X  x  0.95 [A.U A/M 2017]

Solution :

Given that

P X  x  0.95 and exponential distribution is e  x

1  P X  x  0.95


1   e x dx  0.95
x


1   10e 10 x dx  0.95
x
75
Page

MAILAM ENGINEERING COLLEGE DEPARTMENT OF MATHEMATICS


PROBABILITY &RANDOM PROCESSES- MA6451 RANDOM VARIABLE UNIT-1


 e10 x 
1  10    0.95
 10  x


1  e 10 x 

x  0.95

 e 10 x  0.05

 10 x  log 0.05

 log 0.05
x
10

 (1.30103)
x
10

x  0.13010

 1 1
31(a)A random variable Y is defined as cos x where X has a uniform p.d.f over   , 
 2 2
find mean and standard deviation? [A.U A/M 2017]

Solution:

Given

 1 1
 1 1 1,   x 
The p.d.f of the random variable over   ,  is f ( x )   2 2
 2 2 0, otherwise


Formula Now E (Y )   yf ( x)dx


1
2
  cos x dx
1

2

1
 sin x  2

    1
2
76

 2 sin  / 2 
Page

 
 

MAILAM ENGINEERING COLLEGE DEPARTMENT OF MATHEMATICS


PROBABILITY &RANDOM PROCESSES- MA6451 RANDOM VARIABLE UNIT-1

2
    0.636
 

E (Y 2 )  y
2
f ( x) dx


1
2
  cos 2 x dx
1

2


2
1  cos 2x dx
 1 2

2

1
 x sin 2x  2
 
2 2   1
2

 1 sin    1 sin(  ) 
    
4 4   4 4 

1
 
2

Var (Y )  E (Y 2 )  E (Y )
2

 0.5  0.636
2

 0.096

S .D  0.096  0.31

31(b) A manufacturer produces airmail envelopes whose weight is normal with mean
  1.950 gm and S.D.   0.025 gm. The envelopes are sold in lots of 1000. How many
envelopes in a lot may be heavier than 2 grams? [A.U A/M 2017]

Solution:

Given   1.950 gm and S.D.   0.025 gm.


77
Page

Let X be a R.V which denotes the weight of an envelope

MAILAM ENGINEERING COLLEGE DEPARTMENT OF MATHEMATICS


PROBABILITY &RANDOM PROCESSES- MA6451 RANDOM VARIABLE UNIT-1

Formula The standard normal variate

X   X  1.950
z 
 0.025

2  1.950
When X=2, z  2
0.025

P (the envelopes heavier than 2)= P ( X  2)

 P ( Z  2)

 0.5  P (0  Z  2)

 0.5  0.4772

 0.0228

Out of 1000, the number of envelopers heavier than 2  1000 * P ( X  2)

 1000 * 0.0228

 23

32(a).A car hire firm has 2 cars . The number of demands for a car on which day
distributed as Poisson variate with mean 0.5.calculate the proportion of days on which (i)
Neither car is used (ii) Some demand is refused. [A.U A/M 2017]
Solution:

Given : mean=   1.5

Let X denote the no. of breakdown of a computer

e   x
Formula The probability of distribution is P( x  x ) 
x!

x
e 0.5  0.5 
P ( x  x) 
x!

0
e 0.5  0.5 
(1) P(neither car is used) = P ( x  0)   0.606  61%
0!
78
Page

MAILAM ENGINEERING COLLEGE DEPARTMENT OF MATHEMATICS


PROBABILITY &RANDOM PROCESSES- MA6451 RANDOM VARIABLE UNIT-1

Proportion of days on which some demand is refused

 P ( X  2)
 1  P (X  2)
 1  [P (X  0 )  P (X  1)  P (X  2)]
 e  0 .5 (0 .5) 0 e  0 .5 (0.5) 1 e  0 .5 (0 .5) 2 
1    
 0! 1! 2! 
 0.01 6  2 %

32(b) In a test on 2000 electric bulbs ,it was found that the life of a particular make, was
normally distributed with an average life of 2040 hours and S.D. of 60 hours. Estimate the
number of bulbs likely to burn for (i) more than 2015 hours (ii) less than 1950 hours (iii)
more than 1920 hours but less than 2160 hours [A.U N/D 2017]

Solution
Given:   2040,   60,
X 
Formula: Z 

(i) P(Less than 2150 hours)=P(X>2150)
X  2040 2150  2040
 Z   1.833
60 60
P  X  2150  P  z  1.833
 0.5  P  0  Z  1.833
 0.5  0.4664
 0.0336
The number of bulbs expected to burn for more than 2150 hours
 2000  0.0336  67
(ii) P(Less than 1950 hours)= P  X  1950
When X  1950
X  2040 1950  2040
 Z   1.5
60 60
P  X  1950   P  z  1.5
 0.5  P  0  Z  1.5
79

 0.5  0.4332
Page

MAILAM ENGINEERING COLLEGE DEPARTMENT OF MATHEMATICS


PROBABILITY &RANDOM PROCESSES- MA6451 RANDOM VARIABLE UNIT-1

P  X  1950 =0.0668
The number of bulbs expected to burn for more than 2150 hours
 2000  0.0668  134
(iii) P(Less than 1920 hours but less than 2160 hours )
= P (1920  X  2160)
X  2040 1920  2040
 Z   2
60 60
When X  2160
X  2040 2160  2040
 Z  2
60 60
P (1920  X  2160)  P  2  Z  2 
 2 P (0  Z  2)
 2  0.4773  0.9546
The number of bulbs  2000  0.9546  1909

RESULT: The number of bulbs  2000  0.9546  1909

ANNA UNIVERSITY QUESTIONS

1 (a). A random variable X has the following function:


X: 0 1 2 3 4 5 6 7

P(X) 0 K 2K 2K 3K K2 2K 2 7K 2  K
:
1
(a) Find K , (b) Evaluate P[X< 6], P[ X  6] , (c) If P[ X  C ]  find the minimum value of
2
(b) C (d) Evaluate P[1.5  X  4.5 / X  2] (e) Find P[ X  2], P[ X  3], P[1  X  5] .

[A.UN/D 2010M/J 2012, M/J2014] .pg.no.18


1.(b)The probability function function of random variable X is defined as
P ( X  0)  3C 2 , P ( X  1)  4C  10C 2 , P ( X  2)  5C  1, where C  0, and P ( X  r )  0 if r  0,1, 2.
find (i) the value of C (iii) The distribution function of X. (iii) The distribution function of X.
(iv) The largest value of X for which F(x)<1/2. pg.no .20 [ A.U M/J 2010]
80
Page

MAILAM ENGINEERING COLLEGE DEPARTMENT OF MATHEMATICS


PROBABILITY &RANDOM PROCESSES- MA6451 RANDOM VARIABLE UNIT-1

1
2. (a). The probability function of an infinite discrete distribution is given by P (X = j) = ; j=1,2,…∞.
2j
Verify that the total probability is 1 and find the mean and variance of the distribution. Find also P(X is
even), P(X≥5) and P(X is divisible by 3).pg.no.21 [A.U N/D 211]

2 (b). If the random variable X takes the values 1,2,3 and 4 such that 2P(X=1) =

3P(X=2) = P(X=3) = 5P(X=4) find the probability distribution and cumulative


distribution function of X. pg.no.242 [AU N/D 2012,A/M 2015]
3. (a). Consider a discrete r. v. ’X’ with probability function

 1

p( X  x)   x( x  1) , x  1,2,3...
0 othewise
Show that E(X) does not exist even though MGF exist.pg.no.23 [ AU N/D 2012]

3.(b) If the probability mass function of a random variable X is given by

 1 5 
p( x)  Kx 3 , x  1,2,3,4 ,find the value of k , p   X   / X  1 ,
 2 2 
Find the mean and variance
.X=x 1 2 3 4
P(X=x) k 8k 27k 64k
Pg.no.25 [A.U M/J 2015]

4. (a) .I f the density function of a continuous random variable X is given by


ax ,0  x  1
a ,1  x  2

f ( x)  
 3a  ax ,2  x  3
 0 , otherwise
(i) Find the value of a
(ii) The cumulative distribution function of X
(iii) If x1,x2 and x3 are 3 independent observations of X. What is the probability that
exactly one of these 3 is greater than 1.5? pg.no.27 [A.U. N/D 2007,N/D 2008]
81
Page

MAILAM ENGINEERING COLLEGE DEPARTMENT OF MATHEMATICS


PROBABILITY &RANDOM PROCESSES- MA6451 RANDOM VARIABLE UNIT-1

4.(b).The Distribution F of a continuous random variable X is given by

0 ,x  0
 1
 x2 ,0  x 
 2
F ( x)   [AU N/D 2011]
 1  3 (3  x ) 2 , 1  x  3
 25 2
 1 ,x  3

Find the p,d,f of X and evaluate P(|X|≤1) and P(1/3<X<4) using both PDF and CDFpg.no29.
2
 x
 2
;x  0
5 (a). If f ( x )   xe , then show that f(x) is a pdf and find F(x).pg no29
 0 ; x  0
[AU N/D 2014]
5. (b) The cumulative distribution function (cdf)of a random variable X is F(x)=1-(1+x)e-x ,x>0.Find the
probability density function of X. Mean and Variance of X. pg.no30
[AU M/J 2006, N/D 2010]
6(a) A continuous random variable X has a pdf f(x)=kx2e-x , x≥0. Find K, rth moment of x
about orgin mean and variance. Pg.no 31 [AUM/J2013]

6 (b). A continuous random variable X has the pdf


K
f ( x)  ;    x   find(i) The value of K,(ii) Distribution function of X,
1  x2
(iii)P[X≥0] pg.no.31
7. (a) The probability density function of a random variable X is given by

x ; 0  X 1

f X ( X )  k ( 2  x);1  X  2 (i) Find the value of K? (ii) Find P0.2  X  1.2
0 ; otherwise

(iii) What is P0.5  X  1.5 / X  1 (iv) Find the distribution function of f(x)
Pg.no32 [A.U. A/M 2011]
7 (b). The cumulative distribution function of the random variable X is given by

 0 , x0

FX ( x)  a (1  x) ,  1  x  1 ; pgno35 [A.U A/M 2015]
 1 , x 1

82

8.(a) A continous random variable X that can assume any value between x=2, x=5 has a
Page

MAILAM ENGINEERING COLLEGE DEPARTMENT OF MATHEMATICS


PROBABILITY &RANDOM PROCESSES- MA6451 RANDOM VARIABLE UNIT-1

f ( x)  k (1  x) .Find P[ X  4] .pg.no 36 [A.U A/M 2015]


density.
1
8 (b). The probability function of an infinite discrete distribution is given by P(X=x)= ,
2x
x=1,2,…...  Find the mean and variance of the distribution. Also find P(X is even).pg.no37

 1  X3
 , x0
9 a). Let ‘X’ be a random variable with p.d.f f ( x)   3 e
0 , othewise

Find (a) P(X>3) (b) MGF of ‘X’ (c) E(X) and Var(X) pg.no39

 1  2x

(b). Let ‘X’ be a random variable with p.d.f f ( x)   2 e , x  0 Find mean and
0 , othewise

variance .pg.no41 [ A.U N/D 2012, 2013 R.P]

 x  2x
 , x0
10 (a). Let ‘X’ be a random variable with p.d.f f ( x)   4 e
0 , othewise

Find mean and variance .pg.no 43 [A.U N/D 2010,2012 2013,M/J 2014 ]

 2 e 2 x , x  0
10.(b) A random variable has the p.d.f given by f  x   
0 , x0
Find (a) the moment generating function.

(b) The first four moment about the orgin. pg.no44 [A.U N/D 2014]

11 (a). Find the mean, variance and MGF of the random variable X having the pdf

x ,0  x  1

f ( x )   2  x ,1  x  2
0 , otherwise

pg.no 45 [ AU N/D 2013]

11(b)Define binomial distribution, Find the moment generating function of binomial distribution and

also find its mean and variance. Pg.no .46 [A.U M/J 14]

12(a).By Calculating the moment generating function of Poisson distribution with parameter  , prove
83

that the mean and variance of the Poisson distribution are equal..pg no.47
Page

MAILAM ENGINEERING COLLEGE DEPARTMENT OF MATHEMATICS


PROBABILITY &RANDOM PROCESSES- MA6451 RANDOM VARIABLE UNIT-1

e x
P  X  x   p ( x)  [ A.U.A/M 2011,N/D 2014,M/J 2015,N/D2015]
x!
12(b) Find the moment generating function, mean and variance of geometric
distribution?pg.no48

13(a) Find the moment generating function of Uniform distribution. Hence find its mean

and variance.pg.no 50

13(b) Define exponential distribution, Find the moment generating function of exponential

distribution and also find its mean and variance. .pg.no 51 [A.U M/J 2015
14(a) Define Gamma distribution, Find the MGF of Gamma distribution and also find mean
and variance. Pg.no 5 3 [A.U N/D 2014,M/J 14,0 2013,2011 ,N/D 2016]
14(b).Find the moment generating function, mean and variance of normal distribution?
Pg.54 [A.U N/D 2014]
15(a) Memoryless Property of Exponential Distribution:pg.no56
15(b) State and prove the memoryless property of the geometric distribution?
Pg.no 56 [A.U M/J 2016, N/D 2015]

16(a) Derive probability mass function of Poisson distribution as a limiting case of binomial
[A.U N/D 2014, 2013 R.P]
distribution. Pg.no 57
16(b) The time (in hours) required to repairs a machine is exponential distributed with
1
Parameter   . [AU N/D 2009]
2
(i) What is the probability that the repair time exceeds 2 2hours?
(ii) What is the conditional probability that a repair takes atleast 10h given that its
duration exceeds 9h?.pg.no58
17(a) Suppose that telephone calls arriving at a particular switchboard follow a Poisson process
with an average of 5 calls coming per minute . What is the probability that up to a minute will elapse
unit 2 calls have come into the switch board? .pg.no59 [AU A/M 2011]
17.(b)Trains arrive at a station at 15 minutes intervals starting at 4a.m.If a passenger arrive at a station
at a time that uniformly distributed between 9.00 and 9.30, find the probability that he has to wait for the
train for (1) less than 6 minutes (2) more than 10 minutes.
Pg.60 [A.U. N/D 2014 P.Q.T]
84
Page

MAILAM ENGINEERING COLLEGE DEPARTMENT OF MATHEMATICS


PROBABILITY &RANDOM PROCESSES- MA6451 RANDOM VARIABLE UNIT-1

18(a)The marks obtained by a number of students for a certain subject is assumed to be normailly
distributed with mean 65 and S.D of 5.If 3 students are taken at random from this set, what is the
probability that exactly 2 of them will have marks over 70.
Pg.no.60 [ A.U A/M 2010,2011]
18(b) ranscendental Meditation(TM) is a continuous random variable X normally distributed with mean
37.6 cc/mm and S.D 4.6 cc/min. Determine the probability that during a period of T.M a person’s oxygen
consumption will be reduced by (i) atleast 44.5 cc/min (ii) atmost 35.0 cc/min (iii) Anywhere from 30.0
to 40.0 cc/min,pg.no 61 [A.U. N/D 2012]
19(a). Let X and Y be independent normal variates with mean 45 and 44 and standard deviation 2 and
1.5 respectively. What is the probability that randomly chosen values of X and Y differ by 1.5 or
more?.pg.no.62 [A.U N/D 2011]
19(b) If X is a uniform random variable in the interval (-2,2) find the probability density function
Y | X | AND E[Y ] pgno62 [P A.U N/D 2011
20(a). If the probability that an applicant for a driver’s license will pass the road test on any given trial is
0.8. what is the probability that he will finally pass the test (1) on the fourth trial and (2) In less tha 4
trials?.pg.no63 [PRP A.U A/M 2010,2015]
20(b). Find the MGF of the two parameter exponential distribution whose density function
 ( xa)
is given by f ( x)   e , x  a and hence find the mean and variance?
Pg.no63 [A.U A/M 2010
21 (a). The marks obtained by a number of students in a certain subject are assumed to be normally
distributed with mean 65 and standard deviation 5. If 3 students are selected at random from this group,
what is the probability that two of them will have marks over 70?
Pg.no 65 [A.U A/M 2010]
21 (b). X is uniformly distributed with mean 1 and variance 4/3, find P X  0
Pg.no.65 [A.UM/J 2015]

22 (a). If X is a uniform random variable in the interval (-1,1) find the probability density
 x 
function Y  sin   .pg.no 66 [ A.U N/D 2011]
 2
2(1  x ) 0  x  1
22 (b). If the pdf of X is given by f ( x)   find its rth moment.
 0 otherwise
85

2
Hence evaluate E[(2 X  1) ] .pg.no66 [ A.U N/D 2012]
Page

MAILAM ENGINEERING COLLEGE DEPARTMENT OF MATHEMATICS


PROBABILITY &RANDOM PROCESSES- MA6451 RANDOM VARIABLE UNIT-1

23(a) A Random variable X is uniformly distributed over (0.10) find p  x  3 , p  x.  7  , px  7  ,


p 2  x  5  .pg.no 67 [A.U M/J 2013]
23 (b) An office has four phones lines .Each is busy about 10% of the time .Assume that the
phone lines act independently (1) What is the probability that all four phones are busy
(ii) What is the probability that atleast two of them are busy.pg.no 68 [ A.U M/J 2013]
24(a). If the probability that an applicant for a driver’s license will pass the road test on any given trial is
0.8. what is the probability that he will finally pass the test (1) on the fourth trial and (2) In less tha 4
trials?pg.no68 [

25(a).In a certain city, the daily consumption of electric power in millions of Kilowatt-hours can be
1
considered as a random variable following gamma distribution with parameters   and   3 .pg.no
2
69
25(b) If the power plant in this city has a daily capacity of 12 million kilowatt-hours, what is the
probability that this supply of power will be insufficient on any given day?
[A.U M/J 2016].pg.no69
26(a).A coin is biased so that a head is twice as likely to appear as a tall. If the coin is tossed
6 times, find the probabilities of getting (1) exactly 2 heads, (2) at least 3 heads, (3)
atmost 4 heads. Pg.no 70 [A.U M/J 2016]
b).The length of time a person speaks over phone follows exponential distribution with
mean 6 mins. What is the probability that the person will talk for(1) more than 8
mins, (2)between 4 and 8 mins. .pg.no 71 [A.U M/J 2016]
27(a).If the probability of success is 1/100 , how many trials are necessary in order that the
probability of atleast one success is greater than 1/2? .pg.no 71 [A.U N/D 2016]

27(b).A and B shoot independently until each has his own target. The probability of their
hitting the target at each shot is 3/5 and 5/7 respectively? Find the probability that B will
require more shots than A. pg.no 71 [A.U N/D 2016]
28(a).If log e X is normally distributed with mean 1 and variance 4, find P (1 / 2  X  2)

given that log e 2  0.693 [A.U N/D 2016]


pg.no 72
28(b). In a normal distribution 31% of items are under 45 and 8% of items are over 64.
86

Find the mean and the standard deviation of the distributionpg.no.73 .[A.U N/D 2015]
Page

MAILAM ENGINEERING COLLEGE DEPARTMENT OF MATHEMATICS


PROBABILITY &RANDOM PROCESSES- MA6451 RANDOM VARIABLE UNIT-1

 2 x,0  x  b
29(a). The p .d. f of a random variable X is given by f ( x ) . For what value of b
0, otherwise

is f(x) a valid pdf? Also find the cdf of the random variable X with the above pdf..pg.no 74

[A.U N/D 2015]


29 b) i) The number of monthly breakdowns of a computer is a random variable having a

Poission distribution with mean equal to 1.8.Find the probability that this computer

Will function for a month (1) Without a break down(2) With only one breakdown.PG 74

30(a) If the probability that a target is destroyed on any one shot is 0.5. What is the

probability that it would be destroyed on 6 th attempt? (A.U N/D 2017) PG-75

Solution:

30(b) Suppose X has an exponential distribution with mean equal to 10. Determine the

 1 1
31(a)A random variable Y is defined as cos x where X has a uniform p.d.f over   ,  find mean and
 2 2
standard deviation? [A.U A/M 2017] PG 76

31(b) A manufacturer produces airmail envelopes whose weight is normal with mean   1.950 gm and

S.D.   0.025 gm. The envelopes are sold in lots of 1000. How many envelopes in a lot may be heavier
than 2 grams? [A.U A/M 2017] PG 77

32(a).A car hire firm has 2 cars . The number of demands for a car on which day distributed as Poisson
variate with mean 0.5.calculate the proportion of days on which (i) Neither car is used (ii) Some demand
is refused. [A.U A/M 2017] PG-78

32(b) In a test on 2000 electric bulbs ,it was found that the life of a particular make, was normally
distributed with an average life of 2040 hours and S.D. of 60 hours. Estimate the number of bulbs likely
to burn for (i) more than 2015 hours (ii) less than 1950 hours (iii) more than 1920 hours but less than
2160 hours [A.U N/D 2017 ] PG 79
87
Page

MAILAM ENGINEERING COLLEGE DEPARTMENT OF MATHEMATICS


PROBABILITY &RANDOM PROCESSES- MA6451 RANDOM VARIABLE UNIT-1

IMPORTANT QUESTIONS

1. Check whether the following is a probability density function or not.

 ke  x , x  0,
f ( x)   [A.U M/J 2016]
0 , elsewhere
(2et  1) 4
2. If a random variable has the moment generating function given by M X (t )  ,
81
determine the variance of X. [AU M/J 2016]
 x2
b
3. If a random variable X is known to a distribution function F ( x)  u ( x)[1  e ],
Where b>0 is a constant. Determine its density function. [A.U N/D 2016]
1
 3 at x  0

2
4. Find the expected value of the distribution where p ( x )   at x  2 [A.U N/D2016]
3


 e  x , x  0,
5. Check whether the following is a probability density function or not. f ( x)  
0 , elsewhere
[AU M/J 2015]

3 2
 (2 x  x ), 0  x  2 , Find
6. A random variable ‘X’ has a p.d.f f ( x )   4 P  X  1 .[AU N/D 2015]
 0 , otherwise
7. A random variable X is uniformly distributed between 3 and 15 .Find the variance of X.
[AU N/D 2015]
8.The mean and variance of binomial distribution are 5 and 4 .Determine the distribution
[AU M/J 2015]
1
9. A random variable ‘X’ has the density function f ( x)  K in    x  
1  x2
Find ‘K’. [AU M/J 20 14]
88

10. A random variable ‘X’ has a p.d.f f ( x )  3 x 2 , 0  x  1 , Find ‘k’ such that
Page

MAILAM ENGINEERING COLLEGE DEPARTMENT OF MATHEMATICS


PROBABILITY &RANDOM PROCESSES- MA6451 RANDOM VARIABLE UNIT-1

P  X  k   0.5
11.X and Y are i.i.d with variance 2 and 3 .Find the variance of 3X+4Y [A.U M/J 2014]
12.A Continuous random variable X has the random variable X has pdf is given by

f ( x)  
a (1 x )2
0 Find a and P  X  4  [A.U M/J 2014]

 x 1  x  1
13. Test whether f ( x)   , can be the pdf of a continuous random variable
 0
[A.U N/D 2014]
14. Define Random variable. [AU N/D 2013]
15. Check whether the following is a probability density function or not.

e  x , x  0,   0
f ( x)   [AU M/J 2012]
 0 , elsewhere
16 Give the Probability law of Poisson distribution and also its mean and variance.
[AU N/D 2011]
17. The cumulative distribution function of the random variable X is given by


 0 , x0
 1 1  1
FX ( x)   x  , 0  x  ; Compute P  X   [AU A/U 2011]
 2 2  4
 1
1, x
 2
18. Let the random variable X denotes the sum of obtained in rolling a pair of fair dice.
Determine the probability mass function of X. [AU A/M 2011]
x 2 3 4 5 6 7 8 9 10 11 12
p(x) 1 2 3 4 5 6 5 4 3 2 1
36 36 36 36 36 36 36 36 36 36 36
2
19. If a random variable has the moment generating function given by M X (t )  ,
2t
determine the variance of X. [AU M/J 2011]
20. What is meant by memoryless property? Which continuous distribution follows this property?
[AU A/M 2010]

1  e  x for x  0
89

21. If a random variable X has the distribution function F ( X )  


 0 for x  0
Page

MAILAM ENGINEERING COLLEGE DEPARTMENT OF MATHEMATICS


PROBABILITY &RANDOM PROCESSES- MA6451 RANDOM VARIABLE UNIT-1

Where  is the parameter, then find P 1  X  2  [AU N/D 2010]


22. Every week the average number of wrong-number phone calls received by a certain mail
Order house is seven. What is the probability that they will receive two wrong calls tomorrow?
[AU N/D 2010]
23. Obtain the mean for a Geometric distribution. [AU A/M 2010]

Ce  2 x ; x  0
24. If the density function of X equals f ( x)   Find C . What is P[X>2]?
0 ;x  0
[AU A/M 2010]

 x2

f ( x)   3 ,  1  x  2 [AU M/J 2006,2009]
25. Show that the function  0 , otherwise

26. If ‘X’ is a continuous random variable whose probability density function is


c( 4 x  2 x 2 ) , 0  x  2
given by f ( x )   What is the value of ‘c’?
 0, otherwise

27. Given that the p.d.f of a R.V ‘X’ is f ( x )  Kx , 0  x  1 , Find K and P  X  0.5 
[AU M/J 2005]
28. A random variable ‘X’ has a p.d.f f ( x )  K , 0  x  1 , Find K . [AU N/D 2005]

 1

29. A random variable ‘X’ has a p.d.f f ( x )   4 , X  2 , Find P  X  1 .
 0, otherwise

 2x , 0  x  1  1
30. A random variable ‘X’ has a p.d.f f ( x)   , Find P X   .
 0 , otherwise  2
[AU M/J 2007]

31. A random variable ‘X’ has a p.d.f f ( x )  3 x 2 , 0  x  1 , Find ‘b’ such that

P  X  b   0.05
[AU M/J 2005]
 x
32. For the following density function f ( x )  ae ,    x   , Find the value of ' a' .

[AU M/J 2006]


90

33. In a continuous random variable ‘X’ having the p.d.f


Page

MAILAM ENGINEERING COLLEGE DEPARTMENT OF MATHEMATICS


PROBABILITY &RANDOM PROCESSES- MA6451 RANDOM VARIABLE UNIT-1

 x2

f ( x)   3 ,  1  x  2 P ( 0  x  1)
 0 , otherwise Find

1
34. A random variable ‘X’ has the density function f ( x)  K in    x  
1  x2
Find ‘K’. [AU N/D 2007,M/J 14]
35. For the following c.d.f. [AU N/D 2004]

0 , x0

F ( x)   x , 0  x  1 , find (i ) P ( X  0.2) (ii ) P (0.2  X  0.5)
 1 , x 1

36. The density function of a random variable ‘X’ is given by
f ( x)  Kx (2  x ) , 0  x  2 Find K.
[AU N/D 2008]

2
 3 at x  1
 1
37. Find the MGF for the distribution where f ( x)   at x  2
 3
 0, otherwise


2
M X (t )  find the variance of ‘X’.
38. If a random variable ‘X’ has the MGF, 2t
[AU M/J 2007]

e  x , x  0
39. Find the MGF of the distribution given by f ( x)  
 0, elsewhere
[AU N/D 2007]
40. State and prove additive property of binomial distribution.

41. Check whether the following data follow a binomial distribution or not. Mean=3;
Variance=4.
[AU M/J 2004]
42. If ‘X’ is a random variate following binomial distribution with mean 2.4 and Variance

find P ( X  5) .
91

1.44, [AU M/J 2003]


43. With the usual notation find ‘p’ for a binomial random variate ‘X’ if n=6 and if
Page

MAILAM ENGINEERING COLLEGE DEPARTMENT OF MATHEMATICS


PROBABILITY &RANDOM PROCESSES- MA6451 RANDOM VARIABLE UNIT-1

9 P ( X  4)  P ( X  2) [AU M/J 2004]

 t
8
44. If the MGF of a r.v. X is of the form 0.4e  0.6 . What is the MGF of 3X+2.
[AU N/D 2007]

45. If is a Poisson variate P ( X  2)  9 P ( X  4)  90 P ( X  6) , find (i) mean of X,


(ii) variance of X.
46. If 3% of the electric bulbs manufactured by a company are defective, find the probability that in a
3
sample of 100 bulbs exactly 5 bulbs are defective (e  0.0498) .
[AU N/D 2002]

47. If the MGF of X is 5  4e 


t 1
, find the distribution of X and c.
[AU N/D 2004]
48. If the probability is 0.05 that a certain kind of measuring device will show excessive drift, what is
the probability that the sixth of these measuring devices tested will be the first to show excessive drift?
49.Find the moment generating function of uniform distribution.
1
50.The time in hours required a machine is exponentially distributed with parameter   , what is the
2
probability that the required time (i) exceeds 2 hours (ii) exceeds 5 hours

51. A random variable ‘X’ has a p.d.f f ( x )  3 x 2 , 0  x  1 , Find ‘k’ such that P  X  k   0.5

[AU M/J 2005,2014]


PART-B
1 (a). A random variable X has the following function:
X: 0 1 2 3 4 5 6 7

P(X) 0 K 2K 2K 3K K2 2K 2 7K 2  K
:
1
(a) Find K , (b) Evaluate P[X< 6], P[ X  6] , (c) If P[ X  C ]  find the minimum value of
2
(b) C (d) Evaluate P[1.5  X  4.5 / X  2] (e) Find P[ X  2], P[ X  3], P[1  X  5] .

[A.UN/D 2010M/J 2012, M/J2014] .pg.no.18


92
Page

MAILAM ENGINEERING COLLEGE DEPARTMENT OF MATHEMATICS


PROBABILITY &RANDOM PROCESSES- MA6451 RANDOM VARIABLE UNIT-1

1.(b)The probability function function of random variable X is defined as


P ( X  0)  3C 2 , P ( X  1)  4C  10C 2 , P ( X  2)  5C  1, where C  0, and P ( X  r )  0 if r  0,1, 2.
find (i) the value of C (iii) The distribution function of X. (iii) The distribution function of X.
(iv) The largest value of X for which F(x)<1/2. pg.no .20 [ A.U M/J 2010]

1
2. (a). The probability function of an infinite discrete distribution is given by P (X = j) = ;
2j
j=1,2,…∞. Verify that the total probability is 1 and find the mean and variance of the distribution. Find
also P(X is even), P(X≥5) and P(X is divisible by 3).pg.no.21 [A.U N/D 211]

2 (b). If the random variable X takes the values 1,2,3 and 4 such that 2P(X=1) =

3P(X=2) = P(X=3) = 5P(X=4) find the probability distribution and cumulative


distribution function of X. pg.no.242 [AU N/D 2012,A/M 2015]
3. (a). Consider a discrete r. v. ’X’ with probability function

 1

p( X  x)   x( x  1) , x  1,2,3...
0 othewise
Show that E(X) does not exist even though MGF exist.pg.no.23 [ AU N/D 2012]

3.(b) If the probability mass function of a random variable X is given by

 1 5 
p( x)  Kx 3 , x  1,2,3,4 ,find the value of k , p   X   / X  1 ,
 2 2 
Find the mean and variance
.X=x 1 2 3 4
P(X=x) k 8k 27k 64k
Pg.no.25 [A.U M/J 2015]

4. (a) .I f the density function of a continuous random variable X is given by


ax ,0  x  1
a ,1  x  2

f ( x)  
 3a  ax ,2  x  3
 0 , otherwise
(i) Find the value of a
93

(ii) The cumulative distribution function of X


Page

(iii) If x1,x2 and x3 are 3 independent observations of X. What is the probability that

MAILAM ENGINEERING COLLEGE DEPARTMENT OF MATHEMATICS


PROBABILITY &RANDOM PROCESSES- MA6451 RANDOM VARIABLE UNIT-1

exactly one of these 3 is greater than 1.5? pg.no.27 [A.U. N/D 2007,N/D 2008]

4.(b).The Distribution F of a continuous random variable X is given by

0 ,x  0
 1
 x2 ,0  x 
 2
F ( x)   [AU N/D 2011]
 1  3 (3  x ) 2 , 1  x  3
 25 2
 1 ,x  3

Find the p,d,f of X and evaluate P(|X|≤1) and P(1/3<X<4) using both PDF and CDFpg.no29.
2
 x
 2
;x  0
5 (a). If f ( x )   xe , then show that f(x) is a pdf and find F(x).pg no29
 0 ; x  0
[AU N/D 2014]
5. (b) The cumulative distribution function (cdf)of a random variable X is F(x)=1-(1+x)e-x ,x>0.Find the
probability density function of X. Mean and Variance of X. pg.no30
[AU M/J 2006, N/D 2010]
6(a) A continuous random variable X has a pdf f(x)=kx e , x≥0. Find K, rth moment of x
2 -x

about orgin mean and variance. Pg.no 31 [AUM/J2013]

6 (b). A continuous random variable X has the pdf


K
f ( x)  ;    x   find(i) The value of K,(ii) Distribution function of X,
1  x2
(iii)P[X≥0] pg.no.31
7. (a) The probability density function of a random variable X is given by

x ; 0  X 1

f X ( X )  k ( 2  x);1  X  2 (i) Find the value of K? (ii) Find P0.2  X  1.2
0 ; otherwise

(iii) What is P0.5  X  1.5 / X  1 (iv) Find the distribution function of f(x)
Pg.no32 [A.U. A/M 2011]
7 (b). The cumulative distribution function of the random variable X is given by

 0 , x0

94

FX ( x)  a (1  x) ,  1  x  1 ; pgno35 [A.U A/M 2015]


 1 , x 1

Page

MAILAM ENGINEERING COLLEGE DEPARTMENT OF MATHEMATICS


PROBABILITY &RANDOM PROCESSES- MA6451 RANDOM VARIABLE UNIT-1

8.(a) A continous random variable X that can assume any value between x=2, x=5 has a
f ( x)  k (1  x) .Find P[ X  4] .pg.no 36 [A.U A/M 2015]
density.
1
8 (b). The probability function of an infinite discrete distribution is given by P(X=x)= ,
2x
x=1,2,…...  Find the mean and variance of the distribution. Also find P(X is even).pg.no37

 1  X3
 , x0
9 a). Let ‘X’ be a random variable with p.d.f f ( x)   3 e
0 , othewise

Find (a) P(X>3) (b) MGF of ‘X’ (c) E(X) and Var(X) pg.no39

 1  2x

(b). Let ‘X’ be a random variable with p.d.f f ( x)   2 e , x  0 Find mean and
0 , othewise

variance .pg.no41 [ A.U N/D 2012, 2013 R.P]

 x  2x
 , x0
10 (a). Let ‘X’ be a random variable with p.d.f f ( x)   4 e
0 , othewise

Find mean and variance .pg.no 43 [A.U N/D 2010,2012 2013,M/J 2014 ]

 2 e 2 x , x  0
10.(b) A random variable has the p.d.f given by f  x   
0 , x  0
Find (a) the moment generating function.

(b) The first four moment about the orgin. pg.no44 [A.U N/D 2014]

11 (a). Find the mean, variance and MGF of the random variable X having the pdf

x ,0  x  1

f ( x )   2  x ,1  x  2
0 , otherwise

pg.no 45 [ AU N/D 2013]

11(b)Define binomial distribution, Find the moment generating function of binomial distribution and

also find its mean and variance. Pg.no .46 [A.U M/J 14]
95

12(a).By Calculating the moment generating function of Poisson distribution with parameter  , prove
Page

that the mean and variance of the Poisson distribution are equal..pg no.47

MAILAM ENGINEERING COLLEGE DEPARTMENT OF MATHEMATICS


PROBABILITY &RANDOM PROCESSES- MA6451 RANDOM VARIABLE UNIT-1

e x
P  X  x   p ( x)  [ A.U.A/M 2011,N/D 2014,M/J 2015,N/D2015]
x!
12(b) Find the moment generating function, mean and variance of geometric
distribution?pg.no48

13(a) Find the moment generating function of Uniform distribution. Hence find its mean

and variance.pg.no 50

13(b) Define exponential distribution, Find the moment generating function of exponential

distribution and also find its mean and variance. .pg.no 51 [A.U M/J 2015
14(a) Define Gamma distribution, Find the MGF of Gamma distribution and also find mean
and variance. Pg.no 5 3 [A.U N/D 2014,M/J 14,0 2013,2011 ,N/D 2016]
14(b).Find the moment generating function, mean and variance of normal distribution?
Pg.54 [A.U N/D 2014]
15(a) Memoryless Property of Exponential Distribution:pg.no56
15(b) State and prove the memoryless property of the geometric distribution?
Pg.no 56 [A.U M/J 2016, N/D 2015]

16(a) Derive probability mass function of Poisson distribution as a limiting case of binomial
[A.U N/D 2014, 2013 R.P]
distribution. Pg.no 57
16(b) The time (in hours) required to repairs a machine is exponential distributed with
1
Parameter   . [AU N/D 2009]
2
(i) What is the probability that the repair time exceeds 2 2hours?
(ii) What is the conditional probability that a repair takes atleast 10h given that its
duration exceeds 9h?.pg.no58
17(a) Suppose that telephone calls arriving at a particular switchboard follow a Poisson process
with an average of 5 calls coming per minute . What is the probability that up to a minute will elapse
unit 2 calls have come into the switch board? .pg.no59 [AU A/M 2011]
17.(b)Trains arrive at a station at 15 minutes intervals starting at 4a.m.If a passenger arrive at a station
at a time that uniformly distributed between 9.00 and 9.30, find the probability that he has to wait for the
train for (1) less than 6 minutes (2) more than 10 minutes.
Pg.60 [A.U. N/D 2014 P.Q.T]
96
Page

MAILAM ENGINEERING COLLEGE DEPARTMENT OF MATHEMATICS


PROBABILITY &RANDOM PROCESSES- MA6451 RANDOM VARIABLE UNIT-1

18(a)The marks obtained by a number of students for a certain subject is assumed to be normailly
distributed with mean 65 and S.D of 5.If 3 students are taken at random from this set, what is the
probability that exactly 2 of them will have marks over 70.
Pg.no.60 [ A.U A/M 2010,2011]
18(b) ranscendental Meditation(TM) is a continuous random variable X normally distributed with mean
37.6 cc/mm and S.D 4.6 cc/min. Determine the probability that during a period of T.M a person’s oxygen
consumption will be reduced by (i) atleast 44.5 cc/min (ii) atmost 35.0 cc/min (iii) Anywhere from 30.0
to 40.0 cc/min,pg.no 61 [A.U. N/D 2012]
19(a). Let X and Y be independent normal variates with mean 45 and 44 and standard deviation 2 and
1.5 respectively. What is the probability that randomly chosen values of X and Y differ by 1.5 or
more?.pg.no.62 [A.U N/D 2011]
19(b) If X is a uniform random variable in the interval (-2,2) find the probability density function
Y | X | AND E[Y ] pgno62 [P A.U N/D 2011
20(a). If the probability that an applicant for a driver’s license will pass the road test on any given trial is
0.8. what is the probability that he will finally pass the test (1) on the fourth trial and (2) In less tha 4
trials?.pg.no63 [PRP A.U A/M 2010,2015]
20(b). Find the MGF of the two parameter exponential distribution whose density function
 ( xa)
is given by f ( x)   e , x  a and hence find the mean and variance?
Pg.no63 [A.U A/M 2010
21 (a). The marks obtained by a number of students in a certain subject are assumed to be normally
distributed with mean 65 and standard deviation 5. If 3 students are selected at random from this group,
what is the probability that two of them will have marks over 70?
Pg.no 65 [A.U A/M 2010]
21 (b). X is uniformly distributed with mean 1 and variance 4/3, find P X  0
Pg.no.65 [A.UM/J 2015]

22 (a). If X is a uniform random variable in the interval (-1,1) find the probability density
 x 
function Y  sin   .pg.no 66 [ A.U N/D 2011]
 2
2(1  x ) 0  x  1
22 (b). If the pdf of X is given by f ( x)   find its rth moment.
 0 otherwise
97

2
Hence evaluate E[(2 X  1) ] .pg.no66 [ A.U N/D 2012]
Page

MAILAM ENGINEERING COLLEGE DEPARTMENT OF MATHEMATICS


PROBABILITY &RANDOM PROCESSES- MA6451 RANDOM VARIABLE UNIT-1

23(a) A Random variable X is uniformly distributed over (0.10) find p  x  3 , p  x.  7  , px  7  ,


p 2  x  5  .pg.no 67 [A.U M/J 2013]
23 (b) An office has four phones lines .Each is busy about 10% of the time .Assume that the
phone lines act independently (1) What is the probability that all four phones are busy
(ii) What is the probability that atleast two of them are busy.pg.no 68 [ A.U M/J 2013]
24(a). If the probability that an applicant for a driver’s license will pass the road test on any given trial is
0.8. what is the probability that he will finally pass the test (1) on the fourth trial and (2) In less tha 4
trials?pg.no68 [

25(a).In a certain city, the daily consumption of electric power in millions of Kilowatt-hours can be
1
considered as a random variable following gamma distribution with parameters   and   3 .pg.no
2
69
25(b) If the power plant in this city has a daily capacity of 12 million kilowatt-hours, what is the
probability that this supply of power will be insufficient on any given day?
[A.U M/J 2016].pg.no69
26(a).A coin is biased so that a head is twice as likely to appear as a tall. If the coin is tossed
6 times, find the probabilities of getting (1) exactly 2 heads, (2) at least 3 heads, (3)
atmost 4 heads. Pg.no 70 [A.U M/J 2016]
b).The length of time a person speaks over phone follows exponential distribution with
mean 6 mins. What is the probability that the person will talk for(1) more than 8
mins, (2)between 4 and 8 mins. .pg.no 71 [A.U M/J 2016]
27(a).If the probability of success is 1/100 , how many trials are necessary in order that the
probability of atleast one success is greater than 1/2? .pg.no 71 [A.U N/D 2016]

27(b).A and B shoot independently until each has his own target. The probability of their
hitting the target at each shot is 3/5 and 5/7 respectively? Find the probability that B will
require more shots than A. pg.no 71 [A.U N/D 2016]
28(a).If log e X is normally distributed with mean 1 and variance 4, find P (1 / 2  X  2)

given that log e 2  0.693 [A.U N/D 2016]


pg.no 72
28(b). In a normal distribution 31% of items are under 45 and 8% of items are over 64.
98

Find the mean and the standard deviation of the distributionpg.no.73 .[A.U N/D 2015]
Page

MAILAM ENGINEERING COLLEGE DEPARTMENT OF MATHEMATICS


PROBABILITY &RANDOM PROCESSES- MA6451 RANDOM VARIABLE UNIT-1

 2 x,0  x  b
29(a). The p .d. f of a random variable X is given by f ( x ) . For what value of b
0, otherwise

is f(x) a valid pdf? Also find the cdf of the random variable X with the above pdf..pg.no 74

[A.U N/D 2015]


29 b) i) The number of monthly breakdowns of a computer is a random variable having a

Poission distribution with mean equal to 1.8.Find the probability that this computer

Will function for a month (1) Without a break down(2) With only one breakdown.PG 74

30(a) If the probability that a target is destroyed on any one shot is 0.5. What is the

probability that it would be destroyed on 6 th attempt? (A.U N/D 2017) PG-75

Solution:

30(b) Suppose X has an exponential distribution with mean equal to 10. Determine the

 1 1
31(a)A random variable Y is defined as cos x where X has a uniform p.d.f over   ,  find mean and
 2 2
standard deviation? [A.U A/M 2017] PG 76

31(b) A manufacturer produces airmail envelopes whose weight is normal with mean   1.950 gm and

S.D.   0.025 gm. The envelopes are sold in lots of 1000. How many envelopes in a lot may be heavier
than 2 grams? [A.U A/M 2017] PG 77

32(a).A car hire firm has 2 cars . The number of demands for a car on which day distributed as Poisson
variate with mean 0.5.calculate the proportion of days on which (i) Neither car is used (ii) Some demand
is refused. [A.U A/M 2017] PG-78

32(b) In a test on 2000 electric bulbs ,it was found that the life of a particular make, was normally
distributed with an average life of 2040 hours and S.D. of 60 hours. Estimate the number of bulbs likely
to burn for (i) more than 2015 hours (ii) less than 1950 hours (iii) more than 1920 hours but less than
2160 hours [A.U N/D 2017 ] PG 79
99
Page

MAILAM ENGINEERING COLLEGE DEPARTMENT OF MATHEMATICS

Potrebbero piacerti anche